117
DR SAIFUL’S NOTES ON MEDICAL & ALLIED HEALTH PROFESSION EDUCATION: STATISTICS & RESEARCH METHODOLOGY Dr. Muhamad Saiful Bahri Yusoff MD, MScMEd

Dr+Saiful's+Notes+on+Medical+%26+Allied+Health+Education+-+Statistics+%26+Research+Methodology

Embed Size (px)

Citation preview

Page 1: Dr+Saiful's+Notes+on+Medical+%26+Allied+Health+Education+-+Statistics+%26+Research+Methodology

DR SAIFUL’S NOTES ON MEDICAL &

ALLIED HEALTH PROFESSION EDUCATION: STATISTICS & RESEARCH

METHODOLOGY

Dr. Muhamad Saiful Bahri Yusoff MD, MScMEd

Page 2: Dr+Saiful's+Notes+on+Medical+%26+Allied+Health+Education+-+Statistics+%26+Research+Methodology

1

Content RESEARCH IN MEDICAL EDUCATION ......................................................2

STEPS IN RESEARCH ................................................................................8

SAMPLING METHOD................................................................................11

SAMPLE SIZE...........................................................................................16

OVERVIEW ON MEDICAL STATISTICS ...................................................20

STUDY DESIGN........................................................................................23

DESCRIPTIVE STATITISTICS..................................................................30

HYPOTHESIS FORMULATION AND TESTING .........................................38

CONFIDENCE INTERVAL.........................................................................43

EXPLORATORY DATA ANALYSIS (NUMERICAL DATA)...........................46

UNIVARIATE ANALYSIS OF NUMERICAL DATA......................................49

UNIVARIATE ANALYSIS OF CATEGORICAL DATA .................................57

CORRELATION & REGRESSION..............................................................61

CORRELATION .....................................................................................61

SIMPLE LINEAR REGRESSION (SLR) ...................................................64

CORRELATION ........................................................................................72

NONPARAMETRIC STATISTICS ..............................................................76

NON-PARAMETRIC TESTS ......................................................................80

STATISTICAL ANALYSIS: WHICH TO CHOOSE?.....................................84

WRITING A RESEARCH PROPOSAL ........................................................99

VARIABLES............................................................................................102

DATA PRESENTATION...........................................................................106

Z-Score & IT’S USES..............................................................................110

t-test ......................................................................................................113

SENSITIVITY & SPECIFICITY................................................................114

Page 3: Dr+Saiful's+Notes+on+Medical+%26+Allied+Health+Education+-+Statistics+%26+Research+Methodology

2

RESEARCH IN MEDICAL EDUCATION

1. Research and types of research:

• How do we develop knowledge?

o Intuitive knowledge (based on “I feel or I think”)

o Authoritative knowledge (based on authorized person view)

o Logical knowledge (based on experience explanation which is

reasonable and logical.)

o Empirical knowledge (based on judgement back up by facts and

usually 90% correct)

• What is research?

o Research is a systematized effort to gain new knowledge – (Redman

& Mory)

o Literally research means search again and again repeatedly.

o Research is an organized and systematic way of finding answers to

questions.

• Research comprises:

o Defining and redefining problems

o Formulating hypothesis or suggested solutions

o Collecting, organizing and evaluating data

o Making deduction and reaching conclusions

o Testing the conclusions to determine whether they fit the

formulating hypothesis (Clifford Woody)

• Types of research

o Basic research and applied research

o Quantitative research and qualitative research

• Qualitative research:

o Ethnography, cognitive anthropology, etc

o Synthetic rather than analytic

o Generally hypothesis generating

o Investigative methods are non-intrusive

o Data are more impressionistic.

Page 4: Dr+Saiful's+Notes+on+Medical+%26+Allied+Health+Education+-+Statistics+%26+Research+Methodology

3

o Research in such a situation is a function of researcher’s insights

and impressions.

Thomas K.Crowl

• Descriptive research

o Include quantitative and qualitative researches.

o Methodologies include observations, surveys, self-report and tests.

o May operates on the basis of hypotheses.

o Deals with naturally occurring phenomena.

• Ethnographic research

o Descriptive and qualitative research.

o Report is detailed verbal description.

o Carried out in natural setting.

o Researcher as participant and observer.

• Survey

o Descriptive

o Quantitative study

• Correlational research

o Investigate the relationship between two or more variables.

o Searching the relationship of variables in natural setting.

Educational Research

History Descriptive Group comparison Correlational

Ethnographic Survey

Experimental Quasi-experimental

Ex Post Facto or Causal-comparative

Page 5: Dr+Saiful's+Notes+on+Medical+%26+Allied+Health+Education+-+Statistics+%26+Research+Methodology

4

• Group comparison research

o Comparing the values of two or more groups of population.

• Experimental research

o Random selection of the individuals forming the groups

Experimental group

Control group

• Quasi-experimental research

o A type of group comparison research.

o Groups are randomly selected.

• Ex Post Facto or Causal-comparative study

o Ex Post Facto in latin is “after the fact”.

o Values of independent variable of two groups are preset (al ready

present).

2. Quantitative/Qualitative Research:

• Deductive

o Begin with a theory and collect data to test.

• Inductive:

o Begin with observations and attempt to explain by generalizing.

• Deductive reasoning

o A type of logic in which one goes from a general statement to a

specific instance.

• Inductive reasoning:

o Involves going from a series of specific cases to a general

statement.

o The conclusion in an inductive argument is never guaranteed.

• Confirmatory

o Experimental

o Quasi-experimental

o Correlational (non-experimental)

• Exploratory

o Qualitative

Page 6: Dr+Saiful's+Notes+on+Medical+%26+Allied+Health+Education+-+Statistics+%26+Research+Methodology

5

3. Qualitative research methods for data collection

• Interviews

• Focus groups

• Survey: open ended questions

• Observations: recorded in field notes

• Document analysis

• What is qualitative data?

o Data in the form of words, rather than numbers, based on:

Asking open ended questions in:

• Interviews

• Group

• Surveys

Examination of documents

• Observation of situations and actions, recorded in fields

notes

• Uses of qualitative data

o Some social sciences e.g

Anthropology

History

Psychology

Sociology

Public health

Policy analysis

Health care evaluation

4. Types of quantitative research design:

• The research design which are commonly used can be divided into

following groups:

o Non experimental design

Post-test design X O1 no control

Pretest-post test design O1 X O2 no control

Page 7: Dr+Saiful's+Notes+on+Medical+%26+Allied+Health+Education+-+Statistics+%26+Research+Methodology

6

Static group comparison X O1 O2 no control

o True experimental design

Pretest-post test control group design

Exp. Group O1 X O2

R.A

(Random Allocation)

Control O3 O4

Post test control group design

Exp. Group X O1

R.A

Control O2

o Quasi-experimental design

Time series O1 X O2 X O3 X O4

No equivalent control group

• Exp group O1 X O2

• No equivalent control group O3 O4

Separate sample pretest post test design

• R.A – Pretest group O1 X

• R.A – Post test group X O2

5. Purpose of Medical Education Research:

• To improve the functioning of educational programmes by providing

information for:

o Decision making

o Evaluating outcomes

o Supporting advocacy for change

o Contributing to the body of knowledge related to concepts and

methods.

Research is like a plant that grows and grows and grows and grows…

Page 8: Dr+Saiful's+Notes+on+Medical+%26+Allied+Health+Education+-+Statistics+%26+Research+Methodology

7

When it is grown, it throws off seeds of all types (basic, applied and

practical) which in turn sprout and create more research projects…

The process continues with all of the new research ‘plants’ throwing off

seeds, creating additional, related research projects of various types…

Soon there is a body of basic, applied and practical research projects

related to similar topics…

And the process goes on and on…

Page 9: Dr+Saiful's+Notes+on+Medical+%26+Allied+Health+Education+-+Statistics+%26+Research+Methodology

8

STEPS IN RESEARCH

1. Preliminary steps:

• Clarifying the purpose

• Formulating the topic

o State your topic idea as a question

o Identify the main concepts or keywords

2. Finding background information

• Critically analyzing information sources

o Initial appraisal

Author

Date of publication

Edition or revision

Publisher

Title of the journal

o Content of analysis

Intended audience

Objective reasoning

Coverage

Writing style

Evaluative review

3. Five steps to write topic for better research

• Think about your topic

• Define your main concepts

• Think synonyms

• Think of broader terms

• Think of narrower terms

4. Steps in research

• Planning

o Formulation of the study objectives

Page 10: Dr+Saiful's+Notes+on+Medical+%26+Allied+Health+Education+-+Statistics+%26+Research+Methodology

9

General objective – what are the purpose of the study

Specific objective – what are the things you want o find in the

study

o Planning of methods

Study population

• Selection and definition

• Sampling

• Sample size

Variables

• Selection

• Definition

• Scales of measurement

Method of data collection

Method of recording and processing

• Preparing for data collection

o Construction of research instrument

o Pretesting the instrument

• Collection the data

• Processing the data

• Interpreting the data

• Writing a report

5. To prioritize a problem and selection of a topic for research, it is helpful to

ask yourself a series of questions and then try to answer each of them

• Is the problem a current one? Does the problem exist now?

• How widespread is the problem? Are many areas and many people

affected by the problem?

• Does the problem effect social groups, such as students, teachers and

patients?

• Does the problem relate to broad social, economic, and health issues,

such as unemployment income maldistribution, the status of women,

education and maternal and child health?

Page 11: Dr+Saiful's+Notes+on+Medical+%26+Allied+Health+Education+-+Statistics+%26+Research+Methodology

10

• Who else is concerned about the problem? Are top government

officials concerned? Are medical doctors or other professionals

concerned?

• Are the resources available?

• Are measures available to solve the problem?

Review your answers to these questions, and ranked the problem and

arrange them according to the ranking.

Problem identification

Information gathering & knowledge building

Research question/ hypothesis formulation

Planning research

Data collection Data processing

Drawing inference

Confirmation or rejection of hypothesis

Data analysis

Dissemination of findings Report writing

Page 12: Dr+Saiful's+Notes+on+Medical+%26+Allied+Health+Education+-+Statistics+%26+Research+Methodology

11

SAMPLING METHOD

1. Sample and Subject:

• A sample is a subset of the population; it comprises some members

selected from it.

• A subject is a single member of the sample (just like an element is a

single member of the population).

2. Population, Element and Population Frame:

• Population refers to the entire group of people, events, or things of

interest that the researcher wishes to investigate.

• An element is a single member of the population.

• Population frame is a listing of all the elements in the population from

which the sample is drawn.

Sampling Method

Non-Probability sampling

Probability sampling

Convenient sampling

Purposive sampling

Judgment sampling

Quota sampling

Unrestricted sampling (Simple random

sampling) Restricted sampling

Systematic sampling

Stratified random

sampling

Cluster sampling

Area sampling

Double sampling

Page 13: Dr+Saiful's+Notes+on+Medical+%26+Allied+Health+Education+-+Statistics+%26+Research+Methodology

12

3. Sampling:

• Sampling is the process of selecting a sufficient number of elements

from the population, so that a study of the properties or

characteristics of the sample make it possible to generalize such

properties or characteristics to the population.

4. Two major types of sampling design:

• Probability sampling (sample picked at random).

o The elements in the population have equal chance or probability of

being selected as sample subjects.

o Probability sampling designs are used when the representativeness

of the samples is of importance in the interests of wider

generalisability.

• Non-probability sampling (sample not randomly picked).

o The elements do not have a predetermined chance of being selected

as subjects.

o When time or other factors, rather than generalisability, become

critical, non probability sampling design are chosen.

5. Probability Sampling:

• Unrestricted sampling:

o More commonly known as simple random sampling.

o Every element in the population has a known and equal chance of

being selected as a subject.

o Advantage:

This kind of sampling method has the least bias.

o Disadvantages:

Cumbersome (difficult) and expensive.

An entirely updated listing (population frame) of the population

may not always available.

• Restricted (complex) random sampling:

o Offer a viable, and sometimes more efficient alternative to the

unrestricted design.

Page 14: Dr+Saiful's+Notes+on+Medical+%26+Allied+Health+Education+-+Statistics+%26+Research+Methodology

13

o Five most common complex probability sampling methods

Systematic sampling

• Drawing every nth element in the population starting with a

randomly chosen between 1 and n.

• For example, if we want a sample of 60 students from total

population of 300 students, we could sample every 9th

student (9, 18, 27, …) until 60 students are selected.

• The number must be selected randomly for example we san

take out one dollar ringgit and choose the last digit of money

number.

Stratified random sampling

• When sub-population vary considerably, it is advantageous

to sample each sub-population (stratum) independently.

• Stratification is the process of grouping members of the

population into relatively homogenous subgroups before

sampling.

• The strata should be mutually exclusive: every element in the

population must be assigned to only one stratum.

• The strata should also collectively exhaustive: no population

element can be excluded.

• The random sampling is applied within each stratum.

Cluster sampling

• Cluster sampling is used when natural grouping are evident

in the population.

• The total population is divided into groups or clusters.

• Elements within a cluster should be heterogenous as

possible.

• But there should be homogeneity between clusters.

• Each cluster must be mutually exclusive and collectively

exhaustive.

• A random sampling technique is then used on any relevant

clusters to choose which clusters to include in the study.

Page 15: Dr+Saiful's+Notes+on+Medical+%26+Allied+Health+Education+-+Statistics+%26+Research+Methodology

14

Area sampling

• One version of cluster sampling is area sampling or

geographically clusters sampling.

• Clusters consist of geographical areas.

• A geographically dispersed population can be expensive to

survey.

• Greater economy than simple random sampling can be

achieved by treating several respondents within a local area

as a cluster.

Double sampling

• A sampling design where initially a sample is used in a study

to collect some preliminary information of interest, and later

a sub-sample of this primary sample is used to examine the

matter in more detail

• It is like reverse pilot study because in double sampling take

all population then proceeds with sampling the interest sub-

sample.

6. Non-probability Sampling:

• The elements in the population do not have any probabilities attached

to their being chosen as sample subjects.

• The findings from the study of the sample cannot be confidently

generalized to the population.

• This method is chosen when generalisability is not critical; focus may

be on obtaining preliminary information in a quick and inexpensive

way.

• 2 broad categories:

o Convenience sampling

Collection of information from members of the population who

are conveniently available to provide it.

o Purposive sampling

Page 16: Dr+Saiful's+Notes+on+Medical+%26+Allied+Health+Education+-+Statistics+%26+Research+Methodology

15

The sampling is confined to specific types of people who can

provide the desired information, either because they are the only

ones who have it, or conform to some criteria set by the

researcher.

2 type of purposive sampling:

• Judgment sampling

o Involves choice of subject who are most advantageously

placed or in the best position to provide the information

required.

o Judgment sampling may curtail the generalisability of the

findings because we are using a sample of experts who

are conveniently available to us.

o Judgment sampling calls for special efforts to locate and

gain access to the individually who do not have the

requisite information.

• Quota sampling

o This method ensures that certain groups are adequately

represented in the study through the assignment of a

quota.

o The quota fixed for each subgroup is based on the total

numbers of each group in the population.

o Considered as a form of appropriateness stratified

sampling, in which a predetermined proportion of people

are sampled from different groups, but on a convenience

basis.

Page 17: Dr+Saiful's+Notes+on+Medical+%26+Allied+Health+Education+-+Statistics+%26+Research+Methodology

16

SAMPLE SIZE

1. Introduction:

• Questions:

o How large should my sample be?

• Answer:

o It depends…

…large enough to be an accurate representation of the

population.

…large enough to achieve statistically significant results.

2. Determining sample size:

• What is sample size that would be required to make reasonably

precise generalizations with confidence?

• A reliable and valid sample should enable us to generalize the findings

from the sample to the population under investigation.

• The sample statistic (statistic finding) should be reliable estimates and

reflect the population parameter (actual finding) as closely as possible

within a narrow margin of error.

• Precision:

o Precision refers to how close our estimate is to the true population

characteristic.

o Normally, the greater the precision required, the larger is the

sample size needed.

• Confidence:

o Confidence denotes how certain we are that our estimate will really

hold true for the population.

o Confidence reflect the level of certainty with which we can state

that our estimates of the population parameters, based on our

sample statistics, hold true.

o Level of confidence can range from 0 to 100%.

o A level of confidence of 95% is conventionally acceptable.

• Sample size is function of…

o Variability (heterogeneity) in the population

Page 18: Dr+Saiful's+Notes+on+Medical+%26+Allied+Health+Education+-+Statistics+%26+Research+Methodology

17

The more variance we find, the bigger the sample should be

o Precision or accuracy needed

The more precise or accurate we want, the bigger the sample

size should be

o Confidence level desired

The higher the confidence level we want, the bigger the sample

size should be

o Type of sampling plan used

Different sampling approaches will require different sample size

• Trade-off between confidence and precision

o If there is little variability in the population, a small sample size

will be sufficient to obtain a high confidence and precision level.

o The higher the precision, the lower will our confidence level be.

o The higher the confidence level, the lower will our precision level

be. That is why, in both cases, we need bigger sample size to

increase the precision and confidence.

• Roscoe proposes the following rules of thumb for determining sample

size

o Sample size larger than 30 and less than 500 are appropriate for

most research

o Where samples are to be broken into sub samples, a minimum

sample size of 30 for each category is necessary.

o In multivariate research (including regression analyses) the sample

size should be several times (preferably 10 times or more) as large

as the number of variables in the study.

o For simple experimental research with tight experimental controls,

successful research is possible with samples as small as 10 to 20

in size.

3. The term statistically significant (p<.05) is used merely as a way

indicating the chances are at least 95 out of 100 that the findings obtained

from the sample of people who participate in the study are similar to what

the findings would be if one were actually able to carry out the study with

the entire population.

Page 19: Dr+Saiful's+Notes+on+Medical+%26+Allied+Health+Education+-+Statistics+%26+Research+Methodology

18

4. Sample size for single mean

n = (Z /∆) 2

If there is a possibility of response from 80% of sample population the

sample size = n/0.8

5. Sample size for two means

n = (A + B) 2 * 22 /∆2

6. Sample size for single proportion:

n = (Z/∆) 2 p (1-p)

7. Sample size for two proportions:

n = (A + B) 2 * [(p1 (1-p1)) + (p2 (1-p2))] / (p1 – p2)2

n = sample size

= population standard

deviation

∆ = precision

Z = Z-score at significance

level

n = sample size

= population standard

deviation

∆ = expected difference of

mean

A = significance level

(usually 95%, equals 1.96)

B = power (usually 80%,

equals 0.84)

Table of values for A and B: Significant level A 5% 1.98 1% 2.58 Power B 80% 0.84 90% 1.28 95% 1.54

n = sample size

∆ = precision

Z = Z-score at significance

level

p = population proportion

n = sample size

A = significance level

(usually 95%, equals 1.96)

B = power (usually 80%,

equals 0.84)

p1 = first proportion

p2 = second proportion

Power is the probability that the null hypothesis will be correctly rejected i.e. rejected when there is indeed a real difference or association. It can also be thought of as “100 minus the percentage chance of missing real effect” – therefore the higher the power, the lower the chance of missing a real effect.

Page 20: Dr+Saiful's+Notes+on+Medical+%26+Allied+Health+Education+-+Statistics+%26+Research+Methodology

19

Some definition:

Sampling error is the difference of statistically finding between actual

parameter of population

Standard error is means of deviation values between two or more groups of

sample or population.

Standard deviation is means of deviation values between two or more units

of samples or population.

Page 21: Dr+Saiful's+Notes+on+Medical+%26+Allied+Health+Education+-+Statistics+%26+Research+Methodology

20

OVERVIEW ON MEDICAL STATISTICS

1. Some introduction:

• I’m interested in research…

• I’m forced to do research…

• Whatever the reason may be…

2. What should I do?

• How should I start???

3. Let’s make it easily understandable

• Research methods/ approaches – leading the way/ direction

• Statistical applications – tools/ vehicles

4. What do I know? Be honest!!

• Do I know about research methods?

o If know back to basic… go back and read research methods/

approaches

• Do I know about statistical and software application?

• Do I know how to interpret?

o OK… I understand methods and approaches

• So… how to proceed?

• Please try to learn medical statistics

• OK… I agree to learn medical statistics

• Tell me how should I go for it (the easiest way)

• Don’t make it complicated (statistician make statistics more difficult)

• Tell me only statistics for non-statisticians

5. Application of statistics in medical research

• Why use statistics?

o Art statistics differences in medical context due to real effects or

random variation or both

Page 22: Dr+Saiful's+Notes+on+Medical+%26+Allied+Health+Education+-+Statistics+%26+Research+Methodology

21

• Modern viewpoint of statistics

o Aid for making scientific decision in the face of uncertainty

o A valuable tool in decision making whenever one is uncertain about

the state of nature

• Statistics in medicine

o Increasingly prevalent in medical practice

Hospital utility statistics, auditing, vaccination uptake,

incidence/prevalence of AIDS and so on…

• Statistics is about common sense and good design

• Statistics is only the guide to make decisions

• Judgment should be made based on both biological and statistical

plausibility

• Concept and applications of statistics in medical sciences

o Let us discuss briefly

o People say “stat is boring”

o Let us make it interesting

6. Classification of statistics

• It consist of two parts

o Descriptive statistics

Concerned with collection, organization, enumeration of the

frequency of characteristics, summarization and presentation of

data.

o Inferential statistics

Statistical inference

Analytical in nature

Consists of a collection of principles or theorems

Allows researcher to generalize characteristics of a “population”

from the observed characteristics of a “sample”

• Statistical jargons

o Population parameter

A fixed numerical value which describes a particular

characteristic of a population

Page 23: Dr+Saiful's+Notes+on+Medical+%26+Allied+Health+Education+-+Statistics+%26+Research+Methodology

22

E.g. 1 – the mean value in the population of a particular

characteristic of interest (mean systolic blood pressure of

Australia adults)

E.g. 2 – the proportion of individuals in the population with a

particular characteristics of interest (the proportion of low birth

weight babies born in Indonesia)

o Sample statistics

Varies in value from sample to sample

Other terms – statistics, summary statistics, point estimate,

effect size, point estimate of the effect size

o The relationship between sample statistics and population

parameters is the basis of statistical inference.

7. Statistical inference

• 2 broad categories

o Hypothesis formulation and testing

o Estimation

Point estimation

Interval estimation

(Confidence interval)

8. Concepts of populations, samples and statistical inference

• Statistical analysis of medical studies is based on the key idea that we

make observations on sample of subjects and then draw inferences

about the populations of all such subjects from which the sample is

drawn.

• If the study sample is not representative of the population we may well

be misled and statistical procedures cannot help

• But even a well designed study can give only an idea of the answer

sought because of random variation in the sample

• Thus result from a single sample is subject to statistical uncertainty,

which is strongly related to the size of the sample. (Gardner MJ and

Altman DG, 1988)

Page 24: Dr+Saiful's+Notes+on+Medical+%26+Allied+Health+Education+-+Statistics+%26+Research+Methodology

23

STUDY DESIGN

1. How do we begin to answer the question?

• Start with the building blocks of any design

o Participants of investigation

o Outcomes of investigation

o Direction of inquiry (prospective or retrospective)

o Other considerations (e.g. possibility and resources)

2. Think about our research question?

• Identify the participants of interest

• What are the outcomes of interest

3. About the investigation

• Presence of a comparison group

o Dependent on the objective of the study

o Generally increases the validity of an observed association

• Exposure (risk) (or intervention) and outcome

o Must be measured with as little error as possible

4. Overview of epidemiologic studies:

Design strategies

Descriptive

Population (prevalence, correlational

studies)

Individual (case report, case series, comparative studies with

historical controls)

Analytical

Observational studies

Intervention studies (experimental; RCT)

Case-control studies

Cohort studies

Cross-sectional studies

Page 25: Dr+Saiful's+Notes+on+Medical+%26+Allied+Health+Education+-+Statistics+%26+Research+Methodology

24

5. Case report

• Strength

o Hypothesis (question) generation

o Clinical observation

• Weaknesses

o May be one off

o Nothing to compare

6. Case series

• Strengths

o Strengthens the hypothesis

o Able to establish temporal relationship

• Weakness

o Nothing to compare

7. Comparative studies with historical controls

• Strength

o Like two case series

o Have something to compare to

• Weaknesses

o May be other differences between groups

o Relies on recoding information being accurate

8. Randomized Controlled Trial

• ‘Goal standard’ test of treatment

• Selection of groups entirely random

• Control group identical to treatment group at start except for

intervention

• Participants/investigators commonly ‘blind’ to group allocation to

reduce bias

• May evaluate good and bad outcomes

Page 26: Dr+Saiful's+Notes+on+Medical+%26+Allied+Health+Education+-+Statistics+%26+Research+Methodology

25

• End point blinding e.g. the pathologist are not given any

information about the study sample slide so the pathologist didn’t

know whose slide it is and he/she will decide based on his/her

independent interpretation about the slide.

• There are few of RCT

o Single blind

o Double blind

o Triple blind

o Multiple blind

o End point blinding

• There are 2 design of RCT

o Parallel RCT

o Cross-over RCT

9. Prospective cohort:

• A group of people (cohort) is assembled, none of whom has

experienced the outcome interest

Population

Eligible subjects

Randomization

Pre-treatment assessment

Pre-treatment assessment

Test Control

Post-treatment assessment

Post-treatment assessment

Parallel RCT

Population

Eligible subjects

Randomization

Test

Outcome assessment

Control

Control Test

Outcome assessment

Cross-over RCT

Page 27: Dr+Saiful's+Notes+on+Medical+%26+Allied+Health+Education+-+Statistics+%26+Research+Methodology

26

• On entry, people are classified according to characteristics that might

be related to outcome

• Other names: longitudinal, prospective, incidence studies.

• Advantages of cohort studies

o The only way of establishing incidence directly

o Follow the same logic as clinical question: if person exposed, do

they get the disease?

o Exposure can be elicited without the bias

o Can assess if the relationship between exposure and many

diseases

o Calculate risk directly: relative risk (RR)

• Strengths

o Powerful design for defining incidence and investigating potential

causes (aetiology questions)

o Establishes temporal sequences

o Appropriate for interventions where can’t randomize

o Investigator has opportunity to measure important variables

completely (not relying on record information)

• Weaknesses

o Expensive and inefficient for rare outcomes – needs more patients

o May be other differences between group

10. Case control studies

• Analytic study design

Exposed

Unexposed

Direction of inquiry

Disease

No Disease

Disease

No Disease

Page 28: Dr+Saiful's+Notes+on+Medical+%26+Allied+Health+Education+-+Statistics+%26+Research+Methodology

27

o Looking back in nature

o We were not there to measure risk directly

o Associate outcome (disease) with prior exposure

• Calculate indirect estimate of risk: odds ratio (OR)

• Compare the frequency of a risk factor in a group of cases and a

group of controls

• There must be a comparison group that does not have the disease

• There must be enough people in the study so that chance does not

play a large part in the observed results

• Groups must be comparable except for the factor of interest

• Advantages of case-control studies

o No need to wait for a long time for disease to occur (causal or

prognostic factors)

o Most important methods used to study rare disease

o Best design for disease with long latent period

o Can evaluate multiple possible potential exposure

• Strengths

o Very efficient design for rare outcomes

• Weaknesses

o Does not allow for the examination of incidence or risk

Cannot directly calculate incidence: OR; and indirect estimate

of risk.

o Increased susceptible to bias in measurement of exposure

Exposure & disease occurred “prior to” the study

Cases

Control

Direction of inquiry

Exposed

Unexposed

Exposed

Unexposed

Page 29: Dr+Saiful's+Notes+on+Medical+%26+Allied+Health+Education+-+Statistics+%26+Research+Methodology

28

• More potential for biases

11. Cross-sectional study

• Distinguish features

o Observe at on particular time or over a period

o Exposure and outcomes measured at the same time

o Information obtained from the subjects only once

• Categories of cross-sectional study

o Descriptive

Prevalence studies

• A point prevalence

o Disease occurrence at the particular time

o E.g. the point prevalence of upper respiratory tract

infection on 1st of July 2005

• A period prevalence

o Disease occurrence at the particular period of time

o E.g. the ten-year year period prevalence (1996-2005) of

the cancer of breast in Malaysia.

o Analytical

Analytical studies is valid only when the current values of the

exposure are extremely stable over time

Two types

Observation

Exposed

Unexposed

Population

Disease

No Disease

Samples

Page 30: Dr+Saiful's+Notes+on+Medical+%26+Allied+Health+Education+-+Statistics+%26+Research+Methodology

29

• Classical cross-sectional

• Comparative cross-sectional study

o A comparative way of conducting a cross-sectional study

o Samples are drawn from two or more defined different

populations

o Measure exposure and outcome factors

o Investigate the association between exposure and

outcome

o Strengths of cross sectional studies

Very quick and inexpensive to implement

Useful for determining prevalence

Appropriate for diagnostic test validity

o Weaknesses

Difficulty in establishing links of causal effect (temporal

relationship)

Impractical for rare outcomes

Page 31: Dr+Saiful's+Notes+on+Medical+%26+Allied+Health+Education+-+Statistics+%26+Research+Methodology

30

DESCRIPTIVE STATITISTICS

1. Definition:

• Statistics

o A field of study concerned with the collection, organization and

summarization of data.

• Statistical Methods

o A scientific technique employed for collection, presentation,

analysis and interpretation of data.

• Biostatistics

o Biological field and medicine

2. Uses of statistical methods

• To collect data in the best possible way

o Designing form

o Organizing

o Conducting survey

• To describe the characteristics of a group or a situation

o Data summary

o Data presentation

• To analyses data and to withdraw conclusion

o Scientific, logic

o Decision making

3. Classification of statistics

• Descriptive statistics

o Concerned with collection, organization, enumeration of frequency

of characteristics, summarization and presentation of data.

o Describe characteristics of the observed data

Type of variable

Summary statistics

Distribution

Graphical presentation

• Inferential statistics

o Analytical in nature

Page 32: Dr+Saiful's+Notes+on+Medical+%26+Allied+Health+Education+-+Statistics+%26+Research+Methodology

31

o Involve hypothesis testing and confidence interval

o Allows researcher to infer/ generalize the characteristics of the

sample (statistic) to the population (parameter)

4. Terms:

• Population:

o Full sets of individuals

o Collection of items objects, things, people

o Parameter – descriptive measure from population data

• Sample

o Subset of population

o Selected to represent the population by sampling technique

o Statistics – descriptive measure from sample data

• Variable

o Any characteristics of even/object/person

o The characteristics being observed/measured

o E.g. age sex, race, height, weight, etc

• Data

o The raw or original measurement of statistics

o Values taken by the characteristics

o E.g. Malay, female, 155cm

5. Classification of variables

Page 33: Dr+Saiful's+Notes+on+Medical+%26+Allied+Health+Education+-+Statistics+%26+Research+Methodology

32

• Discrete

o Characteristics by gaps or interruptions in the values

o Values that can be assume only whole numbers

o Mainly count

o E.g. no of students, no of teeth extracted

• Continuous

o No gap or interruption

o Any value within specified interval

o Mainly measurement

o E.g. height, weight, BP, age, etc

• Nominal

o Unordered categories

o No implied order among the categories

o E.g. race, sex, medical diagnosis, etc.

• Ordinal

o Ordered categories

o Ranked according to some criteria

o E.g. BP – high, normal, low.

Page 34: Dr+Saiful's+Notes+on+Medical+%26+Allied+Health+Education+-+Statistics+%26+Research+Methodology

33

6. Categorical Variables:

• Data presentation

o Statistics

Frequency

Percentage (%)

o Graphical

Pie chart

Bar chart

7. Numerical variables

• Measures of central tendency

o A measure of centrality

Mean

• Arithmetic average

• Adding all the values in a population/sample and divided by

the number of values that are added

• Affected by the extreme value

Page 35: Dr+Saiful's+Notes+on+Medical+%26+Allied+Health+Education+-+Statistics+%26+Research+Methodology

34

Median

• The middle value of data ordered from the lowest to the

highest arrange all value in order

• If n = odd number medical is the middle

• If n = even number median is the mean of 2 middle

observation

• 50th percentile of a set of observation

• The middle value of data ordered from lowest to the highest

value

• Useful for data with non-normal distribution or skewed data

• Less sensitive to extreme values than the mean

• Median (IQR)

Mode

• The most frequent observation

• Point of maximum concentration

• Measures of dispersion/variability

o Range = largest value – smallest value

Different between the largest and smallest value in a set of

observations

Give idea about the variability of data

Simplest to compute

Sensitive to outliers

Least useful

R = Xmax - Xmin

o Variance = s2

Total squares of deviation of observations from the

mean/number of degree of freedom

Average measure of standard deviation of observation from

mean sample

Measures the amount of variability or spread about/from the

mean of a sample

S2 = Σ(xi – xmean)2/n-1

Page 36: Dr+Saiful's+Notes+on+Medical+%26+Allied+Health+Education+-+Statistics+%26+Research+Methodology

35

o Standard deviation (SD)

A square root of variance

The root mean square of the distances (or differences) from

mean of sample

A better measure of variability of a set of data

Smaller SD indicates closer to the mean

Mean (SD)

S = √[Σ(xi – xmean)2/n-1]

o Interquartile range (IQR)

Q3 – Q1

Range between 25th and 75th percentile

Used along with the median

It not affected by outlier

o Percentile = 25th, 50th, 75th, 90th, 95th

8. Normal distribution

• Characteristic

o Bell shaped appearance

o Symmetrical about the mean

o Mean = median = mode

o Total area the curve = 1

o The curve never touch the x line

o SD usually less than 30% of mean value

• Approximately

o 68% 1 SD

Page 37: Dr+Saiful's+Notes+on+Medical+%26+Allied+Health+Education+-+Statistics+%26+Research+Methodology

36

o 95% 2 SD

o 99.7% 3SD

• Mean (SD)

9. Data presentation (numerical data)

• Statistics

o Mean (SD)

o Medical (IQR)

• Graphical

o Histogram

Frequency distribution of quantitative date/continuous data

Bars represent frequency distribution for each class of interval

No spaces between bars

May have equal/unequal class interval

o Box plot

Page 38: Dr+Saiful's+Notes+on+Medical+%26+Allied+Health+Education+-+Statistics+%26+Research+Methodology

37

Histogram

10. Summary

• Categorical data

o Statistics

Frequency (%)

o Graphs

Bar chart

Pie chart

• Numerical data

o Statistics

Mean (SD)

Median (IQR)

o Graphs

Histogram

Box Plot

Page 39: Dr+Saiful's+Notes+on+Medical+%26+Allied+Health+Education+-+Statistics+%26+Research+Methodology

38

HYPOTHESIS FORMULATION AND TESTING

1. Hypothesis:

• A statement about one or more population

• Research question

o Statement

o Research hypothesis

• Postulating the existence of:

o A difference between groups

o An association among factors

• Usually derived from a hunch, an educated guess based on published

results or preliminary observations.

• There are 2 type:

o Null hypothesis (HO)

Hypothesis of no difference

Hypothesis to tested

o Alternative hypothesis (HA)

The hypothesis that postulates that there is a treatment effect,

an association factors or a difference between groups.

• Inferential statistics – estimating the probability that a given outcome

is due to chance

• If the sample data provide sufficient evidence to discredit HO reject

HO in favor of HA.

2. Hypothesis Testing:

• To aid the researcher in reading a decision concerning a population y

examining the sample.

• Observed differences or associations may have occurred by chance.

• HO : the proportion of patients with disease who die after treatment

with the new drug is not different from the proportion of similar

patients who die after treatment with placebo.

Page 40: Dr+Saiful's+Notes+on+Medical+%26+Allied+Health+Education+-+Statistics+%26+Research+Methodology

39

• HA : the proportion of patients with disease who die after treatment

with the new drug is lower than the proportion of similar patients who

die after treatment with placebo.

In the population Statistical Decision

based on a sample HO true HO false

Do not reject HO

Correct (Confidence Limit)

1 –

(level of certainty of our

statistical data hold true)

Type II error

Beta (β) (probability of wrongly not

rejecting HO when the HO

is false)

Reject HO

Type I error (level of significance)

Alpha ()

(probability of wrongly

rejecting the HO when the

HO is true)

Correct (Power of study)

1 – β

(probability that the HO

correctly rejected)

• Type I error ()

o The probability of wrongly rejecting the null hypothesis when the

null hypothesis is true.

• Type II error (β)

o The probability wrongly not rejecting the null hypothesis when

the null hypothesis is false.

• The test statistics

o A value with a known distribution when the null hypothesis is true.

• Normal distribution (refer to descriptive statistics note)

• Level of significant ()

o The null hypothesis is rejected if the probability of obtaining a

value as extreme or more extreme than that observed in the sample

is small when the null hypothesis is true.

o “Small” is usually taken to less than or equal to 5%

o If the 2 tail test is taken then the must be divided by 2

Page 41: Dr+Saiful's+Notes+on+Medical+%26+Allied+Health+Education+-+Statistics+%26+Research+Methodology

40

E.g. = 0.05, p value = 0.05, (2 tail test taken). Significant level

= 0.05/2 = 0.25. Conclusion does not reject the HO.

• The p value

o The probability of obtaining a value as extreme or more extreme

than that observed in the sample given that the null hypothesis is

true is called p value

o The smallest value of for which the null hypothesis can be

rejected

o The p value is compared to the predetermined significance level

(usually 0.05) to decide whether the null hypothesis should be

rejected

o If p value less than , reject the HO.

o If p value greater than , do not reject the HO.

3. Steps in hypothesis testing

• Step 1

o Generate the null hypothesis and alternative hypothesis

HO : ??

HA : ??

What are the characteristics of interest?

• E.g. mean, proportion

• 1-tail (one sided) or 2-tail (both sided)

o E.g. 1-tail research hypothesis

The proportion of patients with disease after treatment

with new drug is lower than the proportion of similar

patients who die after treatment with placebo

o E.g. 2-tail research hypothesis

The mean blood pressure of patients in the new

treatment group is not different from the mean blood

pressure of patients in the old treatment

E.g. research questions:

• Effectiveness of a new antihypertensive drug

Page 42: Dr+Saiful's+Notes+on+Medical+%26+Allied+Health+Education+-+Statistics+%26+Research+Methodology

41

• HO: the mean blood pressure of patients in the new

treatment group is not different from the mean blood

pressure of patients in the old treatment (µ1 = µ2)

• HA: the mean blood pressure of patients in the new treatment

group is different from the mean blood pressure of patients

in the old treatment (µ1 ≠ µ2)

• Step 2:

o Set the significance level

Usually set at 0.05, 0.01, 0.1

• Step 3:

o Decide which statistical test to use and check the assumption of the

test

Population is approximately normally distributed

Data values are obtained by independent random sampling

Adequate sample size

o To decide which statistical test should be used

E.g. mean, proportion

o Assumption must be adequately met

o If not met alternative procedures can be used

E.g. non parametric test would be used when the data is seriously

non-normal)

• Step 4:

o Compute the test statistic and associated p value

Calculate appropriate test statistics

• Step 5:

o Interpretation

Compare p value with the level of significance

Decide whether or not to reject the null hypothesis

p value < – reject the null hypothesis

Notes: 1. in population - µ = mean - = standard deviation

Notes: 2. in sample - x = mean - SD = standard deviation

Page 43: Dr+Saiful's+Notes+on+Medical+%26+Allied+Health+Education+-+Statistics+%26+Research+Methodology

42

p value > – do not reject the null hypothesis

• Step 6:

o Draw conclusions

Conclude accordingly based on rejecting/not rejecting null

hypothesis

o Decision rule

Rejection region

• To reject the null hypothesis if the value of the test statistic that

computed from the sample is one of the values in the rejection

region

Acceptance region

• To accept the null hypothesis of the computed values in the

acceptance region

E.g. conclusion:

• The mean blood pressure of patients in new treatment group is

different from the mean blood pressure of patient in old

treatment

Page 44: Dr+Saiful's+Notes+on+Medical+%26+Allied+Health+Education+-+Statistics+%26+Research+Methodology

43

CONFIDENCE INTERVAL

1. Relationship between confidence interval and hypothesis test

2. Confidence Interval

• Standard Deviation (SD) vs. Standard Error (SE)

o SD – a measure of the variability of individual observation

o SE – a measure of variability of summary statistics

E.g. variability of the sample mean or sample proportion

o SE (SEM) – a special type of standard deviation (the standard

deviation of a sample statistics), depend on

Standard deviation

Sample size

Confidence interval

0.95

Population

Mean (µ)

SD ()

Sample

Sample Sample

Standard Error

Mean (x) SD (s)

Page 45: Dr+Saiful's+Notes+on+Medical+%26+Allied+Health+Education+-+Statistics+%26+Research+Methodology

44

o Sample mean varies from sample to sample (as measured by SE)

o Sample to sample variation of the statistic (sample statistic)

Population

Mean (µ)

SD ()

Sample

How Close?

Lower limit Confidence Interval

Upper limit

Likely to fall

Population parameter

Page 46: Dr+Saiful's+Notes+on+Medical+%26+Allied+Health+Education+-+Statistics+%26+Research+Methodology

45

3. General Comments on Confidence Interval

• As a measure of an estimate of a population parameter (a measure of

the precision of a sample statistic)

• Confidence interval = estimate ± k x (standard error)

• 90% CI, 95% CI, 99% CI

o 95% CI interpretation – 95% certain that the population parameter

lies within its limits.

• Confidence Interval can be calculated:

o Mean

o Relative risk

o Odds ratio

o Hazards ratio

o Correlation coefficient

o Regression coefficient

o Etc…

Width of the CI depend on

SE

Sample size (larger CI narrower more precise estimate)

More variation CI wider Less precise estimate

Page 47: Dr+Saiful's+Notes+on+Medical+%26+Allied+Health+Education+-+Statistics+%26+Research+Methodology

46

EXPLORATORY DATA ANALYSIS (NUMERICAL DATA)

1. Hypothesis test for:

• Single mean

• Difference between two means for independent samples

• Difference between two means (or paired) samples.

2. Single Mean:

• Step 1: Null and Alternative Hypothesis

o H0: The mean serum amylase level in the population from which

the sample was drawn is 120 units/100ml.

o HA: The mean serum amylase level in the population from which

the sample was drawn is different from 120 units/ 100ml. (two-

sided test)

• Step 2: Level of significance

o Alpha = 0.05 (alpha/2 = 0.025)

• Step 3: Check the assumption

o Population is approximately normally distributed

o Random sampling

o Independent variable/sample

• Step 4: Statistical test (one sample t test)

o t = (x - µ0)/ (s/√n)

o where

x = sample mean

s = sample standard deviation

n = sample size

µ0 = the hypothesis mean

t stat has n-1 degrees of freedom

• step 5: interpretation

o p-value < 0.025

o reject H0

• step 6: conclusion

Page 48: Dr+Saiful's+Notes+on+Medical+%26+Allied+Health+Education+-+Statistics+%26+Research+Methodology

47

o The population mean serum amylase is statistically significantly

different from 120 units/ 100ml.

3. Difference between two means for independent samples

• Step 1: Null and Alternative Hypothesis

o H0: The mean serum amylase level in hospitalized and healthy

subjects are the same (µ1 = µ2)

o HA: The mean serum amylase levels in hospitalized and healthy

subjects are different (µ1 ≠ µ2) (two-tailed)

• Step 2: Level of significance

o Alpha = 0.05 (alpha/2 = 0.025)

• Step 3: Check the assumption

o Two population are normally distributed

o Two population have equal variance (Levene’s test)

o Both are independent samples/variables

o Random samples

• Step 4: Statistical test

o Name of the test = independent t-test

o t statistic = (x1 – x2)/√[s2p (1/n1 + 1/n2)]

where: s2p = [ (n1-1)s21 + (n2-1)s22 ]/[n1 + n2 – 2]

• x1, x2 = sample means

• s1, s2 = sample standard deviations

• n1, n2 = sample size

• t stat has n1 + n2 – 2 degree of freedom (df)

o Degree of freedom

o p-value

o 95% confident interval (lower border & upper border)

• Step 5: Interpretation

o P-value < 0.025

o Reject null hypothesis

• Step 6: Conclusion

Page 49: Dr+Saiful's+Notes+on+Medical+%26+Allied+Health+Education+-+Statistics+%26+Research+Methodology

48

o At the 5% level of significance that the mean serum amylase levels

are different in healthy and hospitalized subjects.

4. Difference between two means for dependent (or paired) samples

o Research question: the investigators wanted to determine if treatment

with Amynophylline altered the average number of apneic episodes per

hour

o Step 1: Null and Alternative Hypothesis

o H0: there is no difference in average number of apneic episodes

before and after Amynophylline (no difference = zero)

o HA: the average number of apneic episodes before and after was

difference (difference not equal to zero) (two-tailed)

o Step 2: Level of significance

o Alpha = 0.05 (alpha/2 = 0.025)

o Step 3: Check the assumption

o The population are normally distributes

o The two samples are dependent variables/samples

o Random sampling

o Step 4: Statistical test (paired t test)

o t = d/sd √(n)

o where

d = means of differences

sd = standard deviation of the differences

n = number of pairs

t stat has n-1 degree of freedom

o Step 5: Interpretation

o p-value < 0.025

o reject null hypothesis

o Step 6: Conclusion

o At the 5% level of significance that the average number of apneic

episodes before and after Amyn0phylline were difference.

Page 50: Dr+Saiful's+Notes+on+Medical+%26+Allied+Health+Education+-+Statistics+%26+Research+Methodology

49

UNIVARIATE ANALYSIS OF NUMERICAL DATA

1. Univariate analysis explores each variable in a data set separately:

• It looks at the range of values

• The central tendency of the values

• It describes the pattern of responsible to the variable

• It describes each variable on its own

2. Univariate analysis

• Categorical variable (e.g. housing)

• Numerical variable (e.g. age)

3. Univariate analysis - Numeric

• Statistical analysis

o Point estimation

Count, min, max, average, median, mode.

o Dispersion

Range, standard deviation, variance, co-variance

Skewness, kurtosis

o Missing value

o Outliers

o Binning

• Visualization

o Histogram, box plot and etc…

Univariate Analysis – Numeric

Age

Count 900 Average 35.25 St Dev 11.20

Min 19 Median 33 Variance 125.37

Max 75 Mode 27 Covariance 32%

Range 55 Skewness 1.09

Missing 0 Kurtosis 0.88

Page 51: Dr+Saiful's+Notes+on+Medical+%26+Allied+Health+Education+-+Statistics+%26+Research+Methodology

50

Univariate Analysis – Challenges

Variable

Categorical Numeric

Missing values Missing values

Invalid values Outliers

Numerization Binning

4. Missing data

• Data entry error

• Data processing error

• Certain data may net be available at the time of entry

• How to handle missing data

o Fill in the missing values manually

o Ignore the records with missing data

o Fill in it automatically

A global constant (e.g. “?”)

The variable mean

5. Outliers

• Data points inconsistent with the majority of data

• Different outliers

o Valid: CEO’s salary

o Noisy: one’s age = 200, widely deviated points

• Removal methods

o Box plot

o Clustering

o Curve-fitting

6. Binning

• Binning is a process of transferring continuous variables into

categorical counterparts

• Binning methods

o Equal-width

Page 52: Dr+Saiful's+Notes+on+Medical+%26+Allied+Health+Education+-+Statistics+%26+Research+Methodology

51

o Equal-frequency

o Entropy-based methods

• Variable values (e.g. age)

o 0, 4, 12, 16, 16, 18, 24, 26, 28

• Equal-width binning

o Bin 1: 0, 4 [-, 10] bin

o Bin 2: 12, 16, 16, 18 [10, 20] bin

o Bin 3: 24, 26, 28 [20, +] bin

• Equal-frequency

o Bin 1: 0, 4, 12 [-, 14] bin

o Bin 2: 16, 16, 18 [14, 21] bin

o Bin 3: 24, 26, 28 [21, +] bin

7. Numerization

• Numerization is the process of transferring categorical variable into

numerical counterparts.

• Numerization methods

o Binary method

o Ordinal method

• Variable values (e.g. housing

o For free, own, rent

• Binary method

o For free: 1, 0, 0

o Own: 0, 1, 0

o Rent: 0, 0, 1

• Ordinal method

o Own: 5

o For free: 3

o Rent: 1

8. Quantification

• Introduction

o To conduct quantitative analysis, responses to open-ended questions

in survey research and the raw data collected using qualitative

methods must be coded numerically.

Page 53: Dr+Saiful's+Notes+on+Medical+%26+Allied+Health+Education+-+Statistics+%26+Research+Methodology

52

o Most responses to survey research questions already are recorded in

numerical format

In mailed and face-to-face surveys, responses are keypunched

into a data file.

In telephone and internet surveys, responses are automatically

recorded in numerically format.

• Developing code categories

o Coding qualitative data can use an existing scheme or one developed

by examining the data.

o Coding qualitative data into numerical categories sometimes can be

a straightforward process

Coding occupation, for example, can rely upon numerical

categories defined by the Bureau of the census.

o Coding most forms of qualitative data, however, requires much effort

o This coding typically requires using an iterative procedure of trial

and error

o Consider, for example, coding responses to the question, “What is

the biggest problem in attending college today?”

o The researcher must develop a set of codes that are;

Exhaustive of the full range of responses

Mutually exclusive (mostly) of one another.

o In coding responses to the question, “What is the biggest problem in

attending college today?” the researcher might begin, for example,

with a list of 5 categories, then realize that 8 would be better, then

realize that it would be better to combine and use a total of 7

categories

o Each time the researcher makes a change in the coding scheme, it is

necessary to restart the coding process to code all responses using

the same scheme

9. Distribution

• Data analysis begins by examining distributions

Page 54: Dr+Saiful's+Notes+on+Medical+%26+Allied+Health+Education+-+Statistics+%26+Research+Methodology

53

• One might begin, for example, by examining the distribution of

responses to a question about formal education, where responses are

recorded within six categories

• A frequency distribution will show the number and percent of

responses in each category of a variable

10. Central tendency

• A common measure of central tendency is the average or mean of the

responses

• The median is the values in the middle case when all responses are

rank-ordered

• The mode is the most common responses

• When data are highly skewed, meaning heavily balanced toward one

end of the distribution, the median or mode might be better represent

the most common or centered response.

• Consider this distribution of respondent ages:

o 18, 19, 19, 19, 20, 20, 21, 22, 85

• The mean equals 27. But this number does not adequately represent

the common respondent because the one person who is 85 skews the

distribution toward the high end.

• The median equals 20

• This measure of central tendency gives a more accurate portrayal of the

middle of the distribution

11. Dispersion

• Dispersion refers to the way the values are distributed around some

central value, typically the mean.

• The range is the distance separating the lowest and highest values (e.g.

the range of the ages listed previously equals 18-85)

• The standard deviation is an index of the amount of variability in a set

of data

• The standard deviation represent dispersion with respect to the normal

(bell shape) curve

Page 55: Dr+Saiful's+Notes+on+Medical+%26+Allied+Health+Education+-+Statistics+%26+Research+Methodology

54

• Assuming a set of numbers is normally distributed, then each standard

deviation equals a certain distance from the mean.

• Each standard deviation (+1, +2, etc) is the same distance from each

other on the bell-shaped curve, but represents a declining percentage of

responses because of the shape of the curve.

• For example, the first standard deviation account 34.1% of the values

below and above the mean

o The figure 34.1% is derived from probability theory and the shape of

the curve.

• Thus approximately 68% of all responses fall within one standard

deviation of the mean

• The second standard deviation accounts for the next 13.6% of the

responses from the mean (27.2% of all responses) and so on.

• Dispersion measures

o Spread around the mean

Variance – too abstract, a step towards standard deviation

Standard deviation (from mean) – more intuitive

o Standard deviation

Average distance between mean and each value in data set

Translates variance into same scale as mean and all the values

High values are generally bad

• If the responses are distributed approximately normal and the range of

responses is low – meaning that most responses fall closely to the mean

– then the standard deviation will be small

o The standard deviation of professional golfer’s score on a gold course

will be low

o The standard deviation of amateur golfer’s scores on a golf course

will be high

13. Continuous and Discrete Variables

• Continuous variables have responses that form a steady progression

(e.g. age, income)

• Discrete (i.e. categorical) variables have responses that are considered

to be separate from one another (i.e. sex, religious)

Page 56: Dr+Saiful's+Notes+on+Medical+%26+Allied+Health+Education+-+Statistics+%26+Research+Methodology

55

• Sometimes, it is matter of debate within the community of scholars

about whether a measured variable is continuous or discrete

• This issue is important because the statistical procedures appropriate

for continuous-level data, especially as related to the measurement of

the dependent variable.

• Example: suppose one measures amount of formal education within

five categories (less than hs, hs, 2 years vocational/college, college, post

college)

• Is this measure continuous or discrete?

• In practice, five categories seem to be cut off point for considering a

variable as continuous

• Using a seven-point response scale will give the researcher greater

chance of deeming a variable to be continuous.

14. Subgroup comparison

• Collapsing response categories

o Sometimes the researcher might want to analyse a variable by using

fewer response categories than were used to measure it

o In these instances, the researcher might want to collapse one or

more categories into a single category

o The researcher might want to collapse categories to simplify the

presentation of the results or because few observations exist within

some categories

• Collapsing response example

Response Frequency

Strongly disagree 2

Disagree 22

Neither agree nor disagree 45

Agree 31

Strongly agree 1

Page 57: Dr+Saiful's+Notes+on+Medical+%26+Allied+Health+Education+-+Statistics+%26+Research+Methodology

56

One might want to collapse the extreme responses and work with just three

categories

Response Frequency

Disagree 24

Neither agree nor disagree 45

Agree 32

• Handling “Don’t Know”

o When asking about knowledge of factual information (“Does you

teenager drink alcohol?”) or opinion on a topic the subject might not

know much about (“Do school officials do enough to discourage

teenagers from drinking alcohol?”), it is wise to include a “don’t

know” categories as a possible responses.

o Analyzing “don’t know” responses, however can be a difficult task

o The research-on-research literature regarding this issues is complex

and without clear-cut guidelines for decision making

o The decisions about whether to use “don’t know” response categories

and how to code and analyse them tends to be idiosyncratic to the

research and the researcher.

Page 58: Dr+Saiful's+Notes+on+Medical+%26+Allied+Health+Education+-+Statistics+%26+Research+Methodology

57

UNIVARIATE ANALYSIS OF CATEGORICAL DATA

1. Categorical data analysis

• One proportion

o Chi-square goodness of fit

• Two proportion (independent sample)

o Pearson chi-square/ fisher Exact test

• Dependent sample (matched or paired)

o Mc Nemar’s test

• Stratified sampling to control cofounder effect

o Mantel-Haenszel test

2. Two proportion (independent sample) – Pearson Chi-square & Fisher

Exact test.

• To test the association between two categorical variable

• IHD vs. Gender

o Does gender associated with IHD status?

• Result of test

o Not significant no association

o Significant an association

• Step 1: State the hypothesis

o H0: There is no association between gender and IHD

o HA: There is an association between gender and IHD

• Step 2: set the significance level

o How much? – accept the error in estimating the proportion in the

population

o Usually: = 0.05

• Step 3: check the assumption

o Two variables are independent

o Two variables are categorical

o Expected count of less than 5 is > 20% (take fisher exact test) and

if < 20% (take pearson chi-square test).

Expected count = [row total x column total]/grand total

Page 59: Dr+Saiful's+Notes+on+Medical+%26+Allied+Health+Education+-+Statistics+%26+Research+Methodology

58

• Step 4: statistical test

o Chi-square test or

o Fisher exact test

o X2 = Σ (O – E)2 / E

o Chi-square value:

When the difference between observed and expected increase

Value of chi-square increase p-value decrease significant

increase

• Step 5: Interpretation

o p value = 0.123

do not reject H0

o There is no significant association between gender and IHD status.

• Step 6: conclusion

o There is no significance association between gender and IHD status

using Pearson Chi-square tests (p-value = 0.123)

• Data presentation

Table 1: Association between IHD and gender

IHD

Variable Yes

n (%)

No

n (%)

z stat p-value

Gender

Male

Female

15 (60)

20 (80)

10 (40)

5 (20)

2.381 *0.123

* Pearson Chi-square test

3. Two proportion (dependent sample) – Mc Nemar’s test

• Dependent sample (matched or pair sample)

• X2 = (|b+c|) / (b+c)

• Discordant pair

o Is pair of different outcome

o Use to test the difference in the outcome

• Sample of 25 pair patient with breast cancer

SM Live Die

Live a** *b

RM Die *c d**

* Discordant ** Concordant

Page 60: Dr+Saiful's+Notes+on+Medical+%26+Allied+Health+Education+-+Statistics+%26+Research+Methodology

59

o Matched for age

o Undergone

Simple Mastectomy (SM)

Radical Mastectomy (RM)

o Difference of 5-year survival proportion between two group

• Step 1: state the null and alternative hypothesis

o H0: there is no association between type between type of

mastectomy and 5-year survival proportion in patients with breast

cancer

o HA: there is an association between type of mastectomy and 5-year

survival proportion in patients with breast cancer.

• Step 2: set the significance level

o = 0.05

• Step 3: check the assumption

o Categorical data

o Dependent or matched sample

• Step 4: statistical test

o Mc Nemar’s test

• Step 5: interpretation

o p-value = 0.021

reject H0

o there is significant association between type of mastectomy and 5-

year survival proportion in patients with breast cancer.

• Step 6: conclusion

o There is significant association between type of mastectomy and 5-

year survival proportion in patients with breast cancer using Mc

Nemar’s test (p-value = 0.021)

• Data presentation

Page 61: Dr+Saiful's+Notes+on+Medical+%26+Allied+Health+Education+-+Statistics+%26+Research+Methodology

60

Table 2: Association between type of mastectomy and 5-year survival

proportion in patients with breast cancer

Simple mastectomy

Variable Live

n (%)

Die

n (%)

p-value

Radical

Live

Die

13 (%)

9 (%)

1 (%)

2 (%)

*0.021

* Mc Nemar’s test

Page 62: Dr+Saiful's+Notes+on+Medical+%26+Allied+Health+Education+-+Statistics+%26+Research+Methodology

61

CORRELATION & REGRESSION 1. Relationship between two variables

• Are two variables associated each other?

• To what degree (strength) are they associated?

• In which directions is the relationship?

o Positive or negative

• Change in dependent variable that corresponds to change in

independent variable.

o Prediction

Correlation

Regression

- presence of association

- strength (degree) of

association

- direction of association

- prediction

CORRELATION

1. Is a measure of relationship between two numerical variables

- E.g. the relationship between height and weight, the relationship

between cholesterol and blood pressure.

2. Pattern:

- Elliptical pattern – degree of elongation of the ellipses – proportional to

the correlation coefficient.

- Elliptical pattern – indicative of normally distributed variables

Page 63: Dr+Saiful's+Notes+on+Medical+%26+Allied+Health+Education+-+Statistics+%26+Research+Methodology

62

3. Correlation coefficient (r)

- X increase Y increase r = 1 (perfect positive)

- X increase Y decrease r = -1 (perfect negative)

- No linear relationship r = 0

- r

o < 0.25 poor

o 0.26 – 0.50 fair

o 0.51 – 0.75 good

o 0.76 – 1.00 excellent

- r does not imply a cause and effect relationship

- Correlation should be assessed mathematically, not visually.

- r for statistical sample, ρ (rho) for parameter of population.

- Correlation coefficient:

Pearson’s Correlation coefficient

Spearman’e Ranked Correlation

coefficient

- A measure of degree of

straight line relationship

between two numerical

variable

- At least one variable have a

normal distribution

- Correlation coefficient

calculated on the ranks of

the observation of two

variables

- Rank correlation and

Spearman’s correlation –

similar

- Different when the scatter

plot deviates from an

elliptical shape

4. Example: Relationship between height and weight

- Step 1: state the null and alternative hypothesis

• H0: There is no correlation between height and weight

• HA: There is correlation between height and weight (2-tailed)

- Step 2: set significance level

Page 64: Dr+Saiful's+Notes+on+Medical+%26+Allied+Health+Education+-+Statistics+%26+Research+Methodology

63

• = 0.05

- Step 3: check the assumption

• Both numerical variable

• One of the variables has normal distribution

Histogram

Box and Whisker plot

- Step 4: statistical test

• Pearson correlation (if assumption is met)

• Spearman’s correlation (if assumption is not met)

- Step 5: Interpretation

• p-value = <0.001

reject H0

- step 6: conclusion

• There is a significant, positive and excellence correlation between

height and weight (r = 0.88, p < 0.001)

- Checklist for reporting correlation (Figure 1)

• Correlation coefficient – Pearson’s correlation coefficient/

Spearman’s Ranked correlation coefficient

• Actual p-value of correlation coefficient

• Sample size

• Scatter plot

Correlations

height Height weight Weight

Pearson Correlation 1 .878(**)

Sig. (2-tailed) . .000

height Height

N 100 100Pearson Correlation .878(**) 1

Sig. (2-tailed) .000 .

weight Weight

N 100 100** Correlation is significant at the 0.01 level (2-tailed).

Page 65: Dr+Saiful's+Notes+on+Medical+%26+Allied+Health+Education+-+Statistics+%26+Research+Methodology

64

Figure1: A scatter plot showing high positive correlation between

height and weight

SIMPLE LINEAR REGRESSION (SLR)

1. Regression Analysis

• Regression analysis is a statistical tool that utilizes the relation

between variables so that one variable van be predicted from the other

or others

• Linear regression

o Simple (one independent variable (factor) and one outcome)

o Multiple ( more than one factor and one outcome)

• Logistic Regression (dichotomous dependent variables)

150 155 160 165 170 175 180

Height

55

60

65

70

75

80

Wei

ght

n = 100, Pearson’s r = 0.88, p < 0.001

Page 66: Dr+Saiful's+Notes+on+Medical+%26+Allied+Health+Education+-+Statistics+%26+Research+Methodology

65

2. Simple Linear Regression

• Example of research questions

o Does a relationship exist between oral contraceptive and the

incidence of thromboembolism?

o What is the relationship of a mother’s weight to her baby’s birth

weight?

o Relationship between an animal’s pulse rate and the amount

particular drug administered?

• Simple because only one independent variable

• Linear means the relationship between y (dependent/outcome) and x

(independent/factor) variables can be represented by a straight line

• Analysed linear relationship between two quantitative (numerical)

variables

• Involves estimating the equation of a straight line that defines the

relationship between a dependent variable using a given data set

• The method involved is called method of least squares

• We choose a line such that the sum of squares of vertical distances of

all points from the line is minimized (Q = Σ е2i )

• These vertical distances between y values and their corresponding

estimated values on the line are called residuals (ei = yi – ŷi)

• The line thus obtained is called the regression line or the least-squares

line of best fit

3. Regression line (least squares line of best fit)

• Yi = β0 + β1Xi + єi

o Yi is the value of dependent variable when the value of the

independent variable is Xi

o β0 is Y-interception and is constant

o β1 is the slope of the regression line. It is the change in Yi when Xi is

increased by one unit

o β0 and β1 are called regression coefficients

o єi is random error terms, normally distributed, independent, with

zero mean, and constant variance a2

Page 67: Dr+Saiful's+Notes+on+Medical+%26+Allied+Health+Education+-+Statistics+%26+Research+Methodology

66

4. Linear Regression Model

• Relationship

Yi = β0 + β1Xi + єi

5. True Regression line

• The random error term єi the regression equation accounts for the

scattering of the data points about the regression line

• As the mean of the єis is zero, the mean of Yi (at Xi) is:

o E (Yi) = β0 + β1Xi

o The notation E (Yi) means ‘expected value’ of Yi and represents the

mean of Yi

• Not that the mean of y and on x and the relationship is represented by

a straight line

• This equation represents the true regression line

6. Least square estimate

• Time regression line is unknown

• Estimated regression line:

o Ŷ = β^0 + β^1X least square estimate

Ŷ = is estimated mean

β^0 is y-intercept and is constant

• if x = 0, β^0 is the estimated mean value of Y

β^1 is the slope of the regression line. It is the change in Y when X

is increased by one unit.

7. Least Squares (LS)

• “Best Fit” means difference between actual Y values and predicted Y

values are minimum.

Population Y-interception

Population slope

Dependent (outcome/

response) variable Random error

Independent (factor/

explanatory) variable

n n Σ (Yi – Ŷi) = Σ έ2

i i=1 i=1

Page 68: Dr+Saiful's+Notes+on+Medical+%26+Allied+Health+Education+-+Statistics+%26+Research+Methodology

67

• LS minimizes the Sum of the Squared Differences (SSE)

8. Interpretation of Coefficient

• Slope (β^1)

o The change in the estimated mean value of Y when X is increased by

1 unit

If β^1 = 0.05, then the estimated mean cholesterol level (Y)

changes by 0.05 mmol/dl when the age is (X) increased by 1 year.

• Y-intercept (β^0)

o Average value of Y when X = 0

If β^0 = 3.3, then the mean cholesterol level (Y) is expected to 3.3,

when the age (X) is 0 (???)

8. Measures of variation in Regression

• Total variation (Total Sum of Squares (SSTOT))

o Measures variation of observed Yi around the mean Ymean

• Explained variation (Squared Sum of Regression (SSR))

o Variation due to relationship between X & Y

• Unexplained variation (Square Sum of Error (SSE))

o Variation due to other factor

9. Sum of squares

• Total sum of square (SSTOT)

o Measure of total variation in dependent variable Y

o SSTOT = Σ (Yi – Ymean)2 = SSR + SSE

• Regression Sum of square (SSR)

o Measure the variation ‘explained’ by the regression line

o SSR = Σ (Y^i – Ymean)2

• Error Sum of squares (SSE)

o Measures of the ‘unexplained’ variation in Y or the scatter around

the regression line

o SSE = Σ (Yi – Y^i)2

Page 69: Dr+Saiful's+Notes+on+Medical+%26+Allied+Health+Education+-+Statistics+%26+Research+Methodology

68

10. Hypothesis Testing:

• For Simple Linear Regression

o H0: β1 = 0 (no linear relationship)

o HA: β1 ≠ 0 (there is linear relationship)

o Test statistics: t-distribution

o Rejection rule:

Reject H0 if p-value less than 0.05 (assumed )

• For Multi Linear Regression

o H0: β1 = 0 (no linear relationship)

o HA: β1 ≠ 0 (there is linear relationship)

o Test statistics: F-test for ANOVA table:

F = MSR/MSE

MSR = SSR/dfReg

MSE = SSE/dfError

Measure of variation Y Yi Ŷ = β^0 + β^1Xi (unexplained sum of squares) SSE = (Yi – Y^i)2 (total sum of squares) SSTOT = (Yi – Ymean)2 Y^i (explained sum of squares) SSR = (Y^i - Ymean)2 Ymean X Xi

Notes:

X, Y and slope:

• Positive slope, Y increases with increase in X

• Negative slope, Y decreases with increase in X

Page 70: Dr+Saiful's+Notes+on+Medical+%26+Allied+Health+Education+-+Statistics+%26+Research+Methodology

69

o Rejection rule:

Reject H0 if p-value for the F-test less than 0.05 (assumed )

• Assumption

o The errors are normally distributes

o They are independent

o The mean of random error term is equal to zero

o The variance of random error, 2 (sigma square), is constant.

11. How to analyse

• Exploration of the data

o Descriptive

o Scatter plot between two variables

Check for distribution, relationship and outliers

• Fit the square least line (regression line)

o Using least square method

o It is the best fitting straight line trough the data points in a scatter

plot

o It represents the least square equation and estimates the constant

(a) and slope (b) for and β Y^ = a + bx

o It is constructed by using the method of least square – minimizes the

sum of squared deviations of each point from the mean (regression

line)

• Evaluation of model by R2 (R square)

Model Summary

model R R2 Adjusted R2 Std Error of the estimate

1 .592a .350 .338 .9043

a. Predictors: (constant), Time

o R2 = 0.35, meaning that 35% of the total variation in GPA is

explained by the study time

o R2 measures the closeness of fit of the sample regression equation to

the observed values of Y

o It ranges fro 0 to 1

o Is called coefficient of determination

Page 71: Dr+Saiful's+Notes+on+Medical+%26+Allied+Health+Education+-+Statistics+%26+Research+Methodology

70

• Evaluation b

o Evaluation of β using t-statistics Coefficients

Unstandardized Coefficient 95% CI for Model

B Std error

t

Sig. Lower Upper

1 (constant)

Time

1.461

.389

.315

.073

4.639

5.342

.000

.000

.829

.243

2.093

.534

a- dependent variable for GPA

o H0: β1 = 0 (no linear relationship)

o HA: β1 ≠ 0 (there is linear relationship)

o As p value < 0.001, we reject Ho at 5% significance level and have

sufficient evidence to conclude that there is linear relationship

between study time and GPA.

o Positive β means direct relationship

o Estimated Least Square (LS) equation

GPA = constant + b (study time)

GPA = 1.461 + 0.389 (study time)

• Diagnostic checking for assumption

o The assumptions:

The errors are normally distributes

They are independent

The mean of random error term is equal to zero (linearity)

The variance of random error, 2 (sigma square), is constant or

equal

o Model adequacy checks

After obtaining the least square line or fit

Linear model appropriate? ...R2

Investigate model assumption

Diagnostic procedures carried out through examination of

Residuals (difference between the observed value Y and the fitted

ot the predicted value Y at a given value X

Normality

Page 72: Dr+Saiful's+Notes+on+Medical+%26+Allied+Health+Education+-+Statistics+%26+Research+Methodology

71

• Histogram of unstandardized residuals

Linearity

• Plot of unstandardised residuals against unstandardised

predicted values

• Creating residual: go to analyse regression bivariate save

unstandardised residual and predicted values

Let say the assumption is met.

• Interpretation and conclusion

o 35% of the variation in GPA is explained by study time

o There is significant linear association between GPA and study time

o For each 1 hour increase in study, the GPA of a student increase by

0.39

o We are 95% confident that for each 1 hour change in the study tie,

the GPA increase will lie between 0.24 to 0.53

Page 73: Dr+Saiful's+Notes+on+Medical+%26+Allied+Health+Education+-+Statistics+%26+Research+Methodology

72

CORRELATION

1. Introduction

• Correlation is used to measure and describe a relationship between two

variables

• Correlation measure three characteristics of relationship

o The direction of the relationship

Positive

• It means that when value of one variable increase, the

corresponding value of related variable also increases.

Negative

• It means that when value of one variable increase, the

corresponding value of related variable decreases.

Zero correlation (no correlation)

• It means that when values of one variable increases or

decreases independent to the value of other variable

o The form of the relationship

When value of one variable increases, the corresponding value of

related variable increases or decreases until certain value, but

beyond that value there may have change not in the same trend

or may not have any change at all.

o The degree of the relationship

It measure how strong the relationship between the values of two

variables

2. Application of correlation

• Prediction

o If two variables are positively or negatively related to each other,

then by knowing the value of one of these variables it is possible to

predict the corresponding unknown value of the other variable

• Validity

o Validity is them measure that a test truly is measuring what it

claims to measure.

• Reliability

Page 74: Dr+Saiful's+Notes+on+Medical+%26+Allied+Health+Education+-+Statistics+%26+Research+Methodology

73

o Reliability is the measure whether the test instrument produces the

stable, consistent measurements it is used again and again in the

same group of students or people.

• Theory verification

o Theory is a statement that makes a specific prediction about the

relationship between two variables

o This predicted relation can be verified by correlation test

3. Measures of correlation

• Pearson correlation (Pearson product-moment correlation)

o The Pearson correlation measures the degree and the direction of the

linear relationship and is denoted by the letter r (correlation

coefficient)

o r = (degree to which x and y vary together)/(degree to which x and y

vary separately)

o r = (covariability of x and y)/(variabilitiy of x and y separately)

o r = (SP)/√(SSx SSy)

o SP = sum of product of deviation = Σxy – [ (Σx Σy)/n]

o SSx = sum of squared deviation of x = Σxx – [ (Σx Σx)/n]

• Or SSx = Σx2 – (Σx)2/n

o SSy = sum of squared deviation of y = Σyy – [ (Σy Σy)/n]

• Or SSy = Σy2 – (Σy)2/n

• Spearman correlation (Spearman rank-order correlation)

o It is used when the data are of ordinal variable.

o If it is not then data must be ranked

o Rank order the score separately for each variables with 1 for the

smallest score

Case

no.

Score for

variable 1

Score for

variable 2

Ranked for

variable 1

Ranked for

variable 2

1 3 13 1 2

2 5 14 3 3

3 4 12 2 1

Page 75: Dr+Saiful's+Notes+on+Medical+%26+Allied+Health+Education+-+Statistics+%26+Research+Methodology

74

4 6 15 4 4

5 7 16 5 5

o If there are same score for more than one respondents the final rank

for the respondents will be the average of the ranks

Respondent no Score Rank Final rank

1 3 2 2.5

2 5 5 5

3 2 1 1

4 3 3 2.5

5 4 4 4

o The equitation for the spearman calculation

o rs = 1 – (6ΣD2)/[N(N2-1)]

• N is the number of pair (xy)

• D is the difference between each pair (x – y)

• After calculating the value of r or rs, this is to be compared with the

critical value in the correlation table to decide whether there is

significant correlation between the variables.

o Calculated value tabulated value (significant correlation)

o For one-tailed test df = n-1 and for two-tailed test df = n-2

• df = degree of freedom

o coefficient of determination

• this is squared correlation coefficient

• it measures the percentage of variation shared between the two

variables

• r = 0.40

• r2 = 0.16 i.e. 16%

• Point to be remembered

o Correlation is not causation

o Correlation is affected by the range of data

Page 76: Dr+Saiful's+Notes+on+Medical+%26+Allied+Health+Education+-+Statistics+%26+Research+Methodology

75

o Correlation is affected by the outliers

4. Hypothesis tests with the Pearson correlation

• Two-tailed

o Ho = ρ = 0 (no correlation)

o HA = ρ ≠ 0 (there is correlation)

• One-tailed

o Ho = ρ ≤ 0 (there is no positive correlation)

o HA = ρ > 0 (there is positive correlation)

• Reporting correlation

o r = 0.65, n = 30, p-value < 0.01, one tail or two tail,

o r2 = coefficient of determination

5. Summary

• Correlation is a statistical test to assess the relation between two

variables

• Relation can be positive or negative

• Two method of test are Pearson and Spearman methods

• Test is used in prediction of relationship testing validity and reliability

and verifying theories

• Can be calculated manually using different formulas or using computer

statistical package like SPSS

• Correlation does not say about cause and effect relationship

• The correlation coefficient is influenced by the outliers and or range of

data under analysis

Page 77: Dr+Saiful's+Notes+on+Medical+%26+Allied+Health+Education+-+Statistics+%26+Research+Methodology

76

NONPARAMETRIC STATISTICS

1. Nonparametric Statistics (NPS)

• Name nonparametric indicates – no assumption about parameters

(means, variances)

• Require very few assumptions; it is distribution free

• Use median as a measure of central tendency

o Applied when

The data being analysed is ordinal or nominal

In case of interval or ratio scale data when no assumption can be

made about the population probability distribution

o Appropriate foe small samples that are not normally distributed

o Computationally easier

o Less efficient than parametric counter parts

o Loose information by substituting ranks in place of scales

Parametric test Nonparametric test

One-sample t test Sign test for one sample

Paired t-test

1. Sign test for paired samples

2. Wilcoxon Signed-Ranked test for

pair samples

Two independent sample t-test Man-Whitney test (Wilcoxon Rank

Sum Test)

One-way ANOVA Kruskal-Wallis Test

ANOVA (randomized block design) Friedman’s Test

Pearson’s correlation coefficient Spearman’s Rank correlation

coefficient

2. Sign Test for Matched

• Observation are matched pairs but assumption underlying the paired t-

test are not met, or the measurement scale is weak then Sign Test can

be applied

Page 78: Dr+Saiful's+Notes+on+Medical+%26+Allied+Health+Education+-+Statistics+%26+Research+Methodology

77

• Hypothesis

o H0: ∆d = 0 (the median of differences is zero)

o HA: ∆d ≠ 0

• T.S.: smallest of n+ and n-

• RR: Reject H0 if p-value is less than (assumed alpha)

• Procedure

o Exclude the observations for which the difference (di) is zero

o For di > 0 assign (+sign) and for di < assign (-sign)

3. Wilcoxon Signed-Rank Test for paired samples

• It is sophisticated than Sign test

• Sign test only tell whether the sign of a difference is positive or negative

• This test makes use of both the signs and magnitudes of the differences

• Thus for a strong measurement scale the sign test may be undesirable

since it would not make full use of the information contained in the

data.

• Assumption

o The distribution of difference is continuous

o The distribution of differences is symmetric

• Hypothesis

o H0: ∆d = 0 (the median of differences is zero)

o HA: ∆d ≠ 0

• T.S: T = min (T+, |T-|)

• Rejection region

o Reject H0 if T ≤ critical value or

o Reject H0 if p-value is less than (assumed alpha)

• Procedure

o Calculate the differences of each pair of observations (di)

o Ignore the signs of these differences

o Rank the absolute values from smallest to largest

o Assign the signs of the corresponding differences to these ranks

Page 79: Dr+Saiful's+Notes+on+Medical+%26+Allied+Health+Education+-+Statistics+%26+Research+Methodology

78

o A difference of zero is not ranked, it is eliminated from the analysis

and the sample size is reduced by one

o Tied observation are assigned an average rank (suppose two smallest

differences; 4,4; each one will get average rank (1+2)/2 = 1.5)

o Assign each rank either a (+) or (-) sign corresponding to the sign of

the difference

o Compute sum of +ve ranks (T)+ and sum of –ve ranks (T-)

o Choose the test statistics (smallest of T+, |T-|)

4. Wilcoxon Rank Sum Test (Mann-Whitney-U test)

• Counter part of t-test for two independent samples

• Assumptions

o The two samples have been drawn independently and randomly from

their respective populations.

o The measurement scale is at least ordinal.

o The distributions of the two populations have the same general

shape. They differ only with respect to their medians.

• Hypothesis

o H0: the two populations are identical (∆1 = ∆2)

o HA: population 1 and 2 have different medians (∆1 ≠ ∆2)

• Rejection rule:

o Reject H0 if p-value less than 0.05 (assumed )

• Procedure

o Select independent random samples from each population

o Combine the two samples

o Jointly rank the combined samples. If tied observation, assign an

average to all with the same value

For example: if two observations are tied for the rank 3 and 4

each is given 3.5.

Next higher value receives a rank of 5 and so on

o Label sample smaller sample size as sample 1. test statistic is the

SUM of RANKS for sample 1, denote region from the table

o Determine rejection region from the table

Page 80: Dr+Saiful's+Notes+on+Medical+%26+Allied+Health+Education+-+Statistics+%26+Research+Methodology

79

5. Kruskal-Wallis Test

• Counter part of One Way Analysis of Variance (ANOVA: comparing

means of more than two groups) if:

o Normality assumption of ANOVA not justified

o Or the data available is ordinal (consist ranks)

• Assumption:

o The samples are independent and random

o The measurement scale is at least ordinal

o The distribution of the values is sampled populations are identical

except for the possibility that one or more of the population are

composed of values that tend to be larger than those of other

populations.

• Hypothesis

o H0: the two populations are all identical

o HA: At least one of the population tend to exhibit larger values than

others

• Procedure

o If no ties or moderate number of ties the formula simplifies to:

• Rejection region

o When the samples sizes are large (ni ≥ 5) the test statistic T is

distributed approximately as x2 (t – 1)

o Reject H0 if T > x2 (t – 1)

6. Spearman’s Rank correlation coefficient

• Nonparametric alternative of Pearson’s coefficient of correlation

• Relevant when the measurement scale is at least ordinal or the

relationship between two variables is not linear

• It is denoted by rs

• rs = 1 implies strictly increasing monotonicity

• rs = -1 implies strictly decreasing monotonicity

Page 81: Dr+Saiful's+Notes+on+Medical+%26+Allied+Health+Education+-+Statistics+%26+Research+Methodology

80

NON-PARAMETRIC TESTS

1. Introduction

• Inferential statistics where population parameters are not a requirement

to calculate its value

• A process which is carried out in order to find out whether or not a

particular statistical hypothesis is likely to be true

• A statistical test in which no assumption are made about any statistical

parameter. This is similar to a test in which we do not assume that the

data have any particular distribution

2. The X2 test for goodness of fit

• Test hypothesis about the proportions of a population distribution

• Test how well the sample proportions fit the population proportion

specified by the null hypothesis

• Example:

o Ho : there is no difference in proportion of people in different

categories

o Observed data (fo)

Category 1 Category 2 Category 3

7 26 27

n = 60

o Expected population based on hypothesis (fe)

Category 1 Category 2 Category 3

20 20 20

1/3 of 60 in each category = 1/3 * 60 = 20

o Difference between observed data and expected data (fo – fe)

Category 1 Category 2 Category 3

7 – 20 = -13 26 – 20 = 6 27 – 20 = 7

Page 82: Dr+Saiful's+Notes+on+Medical+%26+Allied+Health+Education+-+Statistics+%26+Research+Methodology

81

o Square the differences (fo – fe)2

Category 1 Category 2 Category 3

(-13)2 = 169 62 = 36 72 = 49

o X2 = Σ [ (square the difference)/(expected data) ]

o X2 = Σ [ (fo - fe)2/(fe) ] chi square formula

o X2 = 169/20 + 36/20 + 49/20

o X2 = 8.45 + 1.8 + 2.45

o X2 = 12.7

o Degree of freedom = number of column – 1

o Degree of freedom = 3 – 1 = 2

3. Chi-square test for independence

• Test a relationship between two variables

• Each individual in the sample is measured or classified on two separate

variables

• Example:

o Ho : there is no relationship between preference of teaching method

and gender of the students

• Variable 1: teaching method

• Variable 2: gender of students

o Observed data (fo)

Teaching Methods Gender

Lecture Tutorial

Total

Male 25 25 50

Female 35 65 100

Total 60 90 150

• 40% students refer lecture

• 60% students prefer tutorial

Page 83: Dr+Saiful's+Notes+on+Medical+%26+Allied+Health+Education+-+Statistics+%26+Research+Methodology

82

o Expected data (fe)

Teaching Methods Gender

Lecture Tutorial

Total

Male *20 30 **50

Female 40 60 100

Total **60 90 ***150

• fe = (**Row total x **Column total)/(***whole total)

• *Example of calculation

o **50 x **60/***150 = *20

o Difference between expected and observed value (fo – fe)

Teaching Methods Gender

Lecture Tutorial

Total

Male 25 -20 = 5 25 – 30 = -5 50

Female 35 – 40 = -5 65 – 60 = 5 100

Total 60 90 150

o Square of difference (fo – fe)2

• 52 = 25, (-5)2 = 25, (-5)2 = 25, 52 = 25

o Formula for X2 = Σ [ (fo - fe)2/(fe) ]

• So X2 = 25/20 + 25/30 + 25/40 + 25/60

• X2 = 1.25 + 0.83 + 0.63 + 0.42 = 3.13

• Degree of freedom = (no. Of column - 1)x(no. of row – 1)

• Degree of freedom = (2-1)x(2-1) = 1

• Critical value = 3.84 when = 0.05 refer to X2 statistic table

o Interpretation

• Calculated values is less than critical value

• Therefore hypothesis is accepted (do not reject the null hypothesis),

there is no difference

o Conclusion

Page 84: Dr+Saiful's+Notes+on+Medical+%26+Allied+Health+Education+-+Statistics+%26+Research+Methodology

83

• There is no relationship between preference of teaching method and

gender of the students

4. Chi-squared test for variance

• This is a test of the null hypothesis that the population variance is 2.

• We have a sample of size n and we compute an unbiased estimate of the

population variance s2 using divisor n-1.

• The distribution used is X2 statistics is (n-1)2 s2/2.

• We assume that the population is normally distributed

• For 95% level of confidence probability level are within 97.5% and 2.5%

• Find out the critical region in MS Excel CHINV (p, df)

Page 85: Dr+Saiful's+Notes+on+Medical+%26+Allied+Health+Education+-+Statistics+%26+Research+Methodology

84

STATISTICAL ANALYSIS: WHICH TO CHOOSE?

1. Process of data management (follow the steps below)

• Research question(s)

• Research design

• Data collection

• Data entry

• Data exploration & cleaning

• Data analysis

• Interpretation

• Writing up

2. Role of statistics in a study

• Statistical knowledge and judgment is required at every step of a study

• What statistical analysis is appropriate to answer the research

question? Points to consider to select the right statistical test:

o Research question/ hypothesis

Are you clear what you want to find out and what design you

have used in your study?

o Number of variables

o Type of data

o Number of groups

o Sample distribution

o Sample type

3. Research question

• The essential question, the study is designed to answer the question

• Most studies are concerned with answering one of four types of

following questions

o What is the magnitude of a health problem or health factor?

o What is the efficacy of an intervention?

o What is the casual relation between one factor (or factors) and the

disease or outcome of interest?

o What is the natural history of a disease?

Page 86: Dr+Saiful's+Notes+on+Medical+%26+Allied+Health+Education+-+Statistics+%26+Research+Methodology

85

• What is/are the research question (s)?

o Common in medical research:

Difference between/ among means

Difference between/ among proportions

Associations between/ among factors

Difference between/ among treatment effects

• Hypothesis

o This is a testable statement that describes the nature of the

proposed relationship between two/ more variables interest

o E.g. there is an association between smoking and coronary heart

disease

4. What is the research design applied and expected result?

• Randomized control trial (RCT)

• Observational studies

o Cross-sectional

o Case-control

o Prospective cohort

o Retrospective cohort

• Case report/ series

• Diagnostic test

• E.g. 1

o Research question: effectiveness of new anti-hypertensive drug

o Research design: randomized controlled trial

Page 87: Dr+Saiful's+Notes+on+Medical+%26+Allied+Health+Education+-+Statistics+%26+Research+Methodology

86

• E.g. 2

o Research question: Risk factor for enteric fever

o Research design: Case control

• E.g. 3

o Research question: maternal & fetal outcome in mother with PIH

o Research design: prospective cohort

Population

(People without disease)

(People with disease)

Time direction

Page 88: Dr+Saiful's+Notes+on+Medical+%26+Allied+Health+Education+-+Statistics+%26+Research+Methodology

87

5. Study factor (s)

• Variable (s) of interest that is hypothesized to be related to health

problem, disease or outcome of interest.

• Also known as the independent variables/ exposure variables/

determinants

6. Outcome factor (s)

• The event or occurrence that is supposed to have as a result of the

study factor

• E.g. the outcome factor is blood pressure, as it influenced by study

factors, salt intake.

• Also known as dependent variable.

7. Number of variables

• One independent variable only – univariate analysis

• More than one independent factor variables – multivariate analysis

• Less likely to conduct and conclude a study with only focusing on

univariate analysis in health sciences

• If there is a multi-factorial effect on the outcome, univariate gives

misleading results

• Example risk factors fro coronary heart disease

• Multivariate analysis can eliminate confounding effect

8. Type of data

• Numerical

o Continuous (e.g. weight)

Population (no disease)

Time direction

Page 89: Dr+Saiful's+Notes+on+Medical+%26+Allied+Health+Education+-+Statistics+%26+Research+Methodology

88

o Discrete (e.g. number of patients admitted)

• Categorical

o Nominal (e.g. occupation, gender)

o Ordinal (e.g. disease severity, socioeconomic status)

• Statistical tests applied are different based on the type of variables

(must consider both independent and dependent variables)

9. Number of group

• Two group (two levels) (e.g. diabetic and non-diabetic group)

• More than two group (more than two levels) (e.g. race – Malay, Chinese,

Indian, Others)

10. Sample distribution

• Normal distribution parametric test

• Non-normal distribution non-parametric test

• Suggested procedure for assessing normality

o Compare the mean & median (for normal distribution mean =

median)

o Construct a histogram overlaid with normal curve

o Construct a box and whisker plot

o Statistical test

Kolmogorov-Sminov test

Shapiro-wilk test

• Non-parametric test are appropriate when:

o Data is ordinal

o Data is non-normal distribution and cannot be easily transformed

o Data may contain outlier

• Non-parametric methods have two general limitations

o Not as powerful as parametric counterparts

o Test for complex design are not readily available in standard

computer packages

11. Sample type

• Independent sample (e.g. disease and non-disease groups, male and

female)

Page 90: Dr+Saiful's+Notes+on+Medical+%26+Allied+Health+Education+-+Statistics+%26+Research+Methodology

89

• Dependent/ paired/ matched sample (e.g. difference of blood pressure

measurements before and after treatment, age and sex matched

samples)

12. What to be asked before choosing a statistical test?

• What is the research question/ hypothesis?

• What is the outcome factor and what are the study factors?

• How many variables?

• How many groups?

• What is the distribution like?

• Are the samples independent?

• Is the data numerical/ categorical?

13. Data exploration and cleaning

• Compulsory to do

• Do not rush to analyze data

• Clean and explore first

• Get acquaintance with the data

• Check duplications

• Out-of-range values and location of error

• Distribution of variables

• Missing data checking consistency errors

• Exploring the relationship between variables

• Transformations

• To get acquaintance with data set before the major analysis is carried

out

o Read the protocol again

o Recall the objectives

o Identify major outcome, exposure and potential confounders/ effect

modifier

o To check records with duplicating ID number (to prevent repeated

data entry

• Error checking

o Respondent’s mis-marking answers

Page 91: Dr+Saiful's+Notes+on+Medical+%26+Allied+Health+Education+-+Statistics+%26+Research+Methodology

90

o Coder’s miscoding response

o Marking errors by data personnel

• Out-of-range values and location errors

o Measurement error

o Recording error

o Genuine observation

• What to do?

o Check again original measurements where possible

o If original measurements suspicious repeat the measurement

o If not possible to check common sense

o If the value is impossible/implausible justifiable to set as

“missing”

14. Distribution of the variables

o Examine each variable

Continuous

• Normal distribution

• If not

o ? transformation

o ? categorization

Categorical

• Frequency distribution

15. Missing data

o Occur when respondent would/could not answer

o Too much missing data

Threat the study

Indicate a problem with a question

o Should not be entered as a blank as some statistical packages

interpret blanks as zeroes

o Common practice – coded as 9, 99 or 999

16. Consistency errors

o Situations where respondents answered a question for which they

were ineligible or when codes were entered incorrectly

o Countercheck with questionnaire/data collection form

Page 92: Dr+Saiful's+Notes+on+Medical+%26+Allied+Health+Education+-+Statistics+%26+Research+Methodology

91

o Can be prevented by proper programming in some statistical

software

17. Exploring the relationship between variables

• Cross tabulation useful for categorical variables (sometimes better to

categorize)

• Should consider confounding & interaction

• Graphs – mostly for continuous variables

• Relationship between the outcome variable and other variables

o E.g. scatter plot

18. Transformation

• Severely skewed data – two approaches

o Use nonparametric methods

o Apply transformation

• Many distributions in medicine – skewed to the right

• Involve performing a mathematical operation on every value of the

variable

• Improves the symmetry of the distribution

Transformation Name Effect

X3 Cube Reduce extreme skewness to left

X2 Square Reduce skewness to left

X1/2 Square root Reduce mild skewness to right

log10 (X) Log Reduce skewness to right

-1/√X -ve reciprocal root Reduce extreme skewness to right

-1/X -ve reciprocal Reduce events more extreme

skewness to right.

• Check the symmetry of the distribution after transformation

• If sufficiently improved use the transformed data

• If resistant to transformation use nonparametric methods

19. Interpretation

• Most confusing part of researchers

Page 93: Dr+Saiful's+Notes+on+Medical+%26+Allied+Health+Education+-+Statistics+%26+Research+Methodology

92

• May be the most difficult part for those who are not familiar with

statistical applications

• Should interpret only when considered to be results of final analysis

stage

o E.g. in multivariate analysis, final model should be interpreted for

writing regardless of the prior more-favorable results towards the

hypothesis

• Recall statistical theory and concepts whenever applicable

• May need help from a medical statistician

20. Univariate analysis

• Test hypothesis between one independent and one dependent variable

21. Multivariate analysis

• Why we need multivariate analysis?

• Purpose of using multivariate analysis

• Common multivariate analysis methods in health sciences research.

Variables Variables

Independent

Predictor

Explanatory

Dependent

Outcome

Response

Covariates

Confounders

Controls

Effect modifiers

Not the primary interest

Must be recognized

Page 94: Dr+Saiful's+Notes+on+Medical+%26+Allied+Health+Education+-+Statistics+%26+Research+Methodology

93

• Confounding

?

o Distortion of a risk factor-disease relationship brought about by

the association of other factors with both risk factor and disease

o Example of confounding:

?

Risk factor

Disease

Confounder

Physical

activity level

Systolic BP

Age

Page 95: Dr+Saiful's+Notes+on+Medical+%26+Allied+Health+Education+-+Statistics+%26+Research+Methodology

94

• Interaction

?

o Exist when the primary relationship of interest between a risk

factor and a disease is different at different levels of the interaction

factor

o Example of interaction

?

e.g. multivariate analysis

Risk factor

Disease

Interaction factor (effect

modifiers)

Employment in an industry

Lung cancer

Cigarette smoking

Risk of lung

cancer

Years of employment in the industry

Smokers

Non-Smokers

Page 96: Dr+Saiful's+Notes+on+Medical+%26+Allied+Health+Education+-+Statistics+%26+Research+Methodology

95

• Are these two groups comparable?

• What are the role covariates?

22. Purpose Multivariate Analysis

• To statistically adjust the effect on variable Y by change in a

particular variable x when others are controlled

o X1 Y (X2, X3, X4… statistically adjusted for e.g. diet CHD

{smoking and age adjusted for})

• To discover the variable X which has the most influence on outcome

variable Y

• To predict the outcome Y

• Whole ideas of multivariate analysis are “How to separate

independent effect of each X and Y”

• Common multivariate analysis methods in health related sciences

research

• Multivariate models

Surgery

Radiation

Compare outcome

Diet

Smoking

Age CHD

Clinical

Pathology

Demographic

Socio-economic

Cancer prognosis

Page 97: Dr+Saiful's+Notes+on+Medical+%26+Allied+Health+Education+-+Statistics+%26+Research+Methodology

96

• Modeling strategies

MTV

Independent Variable Dependent variable

Multiple linear regression

Multiple logistic regression

Log-linear regression

Survival analysis

> 1

> 1

> 1

> 1

1

1

1

1

Independent variables Dependent variables methods

Continuous

Categorical

Continuous

Continuous/ categorical

Continuous/ categorical

Continuous

Categorical

Categorical

Continuous (survival

time)

Continuous

Multiple linear reg.

Multiple logistic reg.

Multiple logistic reg.

Survival analysis

Log-linear analysis

23. Multivariate analysis General Linear Model (GLM)

• The GLM is a flexible statistical model incorporating analysis involving

normally distributed dependent variables and combinations of

categorical and continuous predictor variables.

• The GLM Univariate model procedure provides regression analysis

and analysis of variance one dependent variable by one or more

factors or covariates

• The GLM Multivariate model procedure provides regression analysis

and analysis of variance for multiple dependent variable by one or

more factor or covariates

• The GLM Repeated Measures procedure provides analysis of variance

when the same measurement is made several times on each subject

or case.

Page 98: Dr+Saiful's+Notes+on+Medical+%26+Allied+Health+Education+-+Statistics+%26+Research+Methodology

97

GLM Independent Variable Dependent variable

Univariate GLM

Multivariate GLM

≥ 1

≥ 1

1

> 1

24. Repeated measures in categorical outcome

• When the dependent variable is a numerical variable

Independent Variable Dependent variable Statistical test

Categorical Numerical Repeated measures ANOVA

(parametric)

Categorical Numerical Friedman test (non-parametric)

• When the dependent variable is a categorical variable

Independent Variable Dependent variable Statistical test

Repeated measure

2 measures

2 outcomes

categories

Mc Nemar’s test

2 measures

3++ outcomes

categories

Test of marginal Homogeneity

3++ measures 2 outcomes

categories

Cochran’s Q test

Repeated measure

with independent

variables

Binary

Ordinal

Multiple

Cross-sectional time series (xt)

Logistic regression

(xt logic)

Page 99: Dr+Saiful's+Notes+on+Medical+%26+Allied+Health+Education+-+Statistics+%26+Research+Methodology

98

Count Loglinear regression

(xt poisson)

General estimating equation

(GEE) model (xtgee)

Page 100: Dr+Saiful's+Notes+on+Medical+%26+Allied+Health+Education+-+Statistics+%26+Research+Methodology

99

WRITING A RESEARCH PROPOSAL

1. Introduction:

• Clear statement of the problems or issue to be analysed and the overall

objective of the proposed research.

• Brief summary of relevant studies and literature describing what has

previously been done and what is currently known about the pattern.

• Concise statement of the rationale behind the proposed approach to the

problem

2. Statement of specific research goals:

• List specific objectives

• List specific hypotheses (if any) to be tested

• List the key variables and how they will be operationally defined

3. Study methodology

• Selection of study population

o Size of study population or sample

o Sampling procedure, if any

o Specification of control population, if any

• Description of the experiment or data collection procedure

o Description of research design

o Description of method and intended research tools

o Description of “interfering” (confounding) variables and how they will

be controlled, or how their effects will be evaluated

o If appropriate, a discussion of pitfalls that might be encountered and

of limitations of the procedure proposed

• Diagram of research design (optional): a diagram is useful foe clarifying

points of research strategy

• Analysis plan

o Specify the kinds of data expected to be obtained

Page 101: Dr+Saiful's+Notes+on+Medical+%26+Allied+Health+Education+-+Statistics+%26+Research+Methodology

100

o Specify the means by which the data will be analysed and

interpreted

• Data processing plan

o Hand tabulation or computer

o Analysis technique: statistical measures

o Use of dummy tables

o Test hypothesis or drive hypothesis to meet the objectives of the

study

4. Significance of the research for both practice and theory

5. Time table (Gantt chart)

• Planning phase

• Construction and development of research instruments

• Pre-testing of research tools and techniques

• Selection of population

• Data collection

• Data preparation (coding, editing, cleaning, etc)

• Data analysis

• Report writing

6. Personnel

• Principal investigator

• Assistants

• Supporting persons

7. Facilities available

• Office space

• Resources in field area

• Data analysis equipment

• Other assistance

Page 102: Dr+Saiful's+Notes+on+Medical+%26+Allied+Health+Education+-+Statistics+%26+Research+Methodology

101

8. Collaboration arrangement

• Describe the collaboration

9. Detailed budget

• Personnel

• Consultant fees

• Supplies

• Travel expenses

• Data processing

• Other expenses

Page 103: Dr+Saiful's+Notes+on+Medical+%26+Allied+Health+Education+-+Statistics+%26+Research+Methodology

102

VARIABLES

1. Types of variables

• Continuous or quantitative variables

• Discrete or qualitative variables

2. Continuous or quantitative variables

• Interval-scale variables

o Interval scale data has order and equal intervals.

o Interval scale variables are measured on a linear scale, and can take

on positive or negative values.

o It is assumed that the intervals keep the same importance

throughout the scale.

o They allow us not only to rank order the items that are measured

but also to quantify and compare magnitudes of differences between

them.

o With interval data, one can perform logical operations, add, and

subtract, but one cannot multiply or divide.

o For instance, if a liquid is at 40 degrees and we add 10 degrees, it

will be 50 degrees. However, a liquid at 40 degrees does not have

twice the temperature of a liquid at 20 degrees because 0 degrees

does not represent ‘no temperature’

• Ratio-scale interval

o Finally, in ratio measurement there is always an absolute zero that

is meaningful.

o This means that you can construct a meaningful fraction (or ratio)

with a ratio variable.

o Weight is a ratio variable.

o In applied social research most ‘count’ variables are ratio, for

example, the number of clients in past six months.

o Why? Because you can have zero clients and because it is

meaningful to say that “…we had twice as many clients in the past

six months as we did in the previous six months.”

Page 104: Dr+Saiful's+Notes+on+Medical+%26+Allied+Health+Education+-+Statistics+%26+Research+Methodology

103

3. Qualitative or Discrete Variables

• Discrete variables is also called categorical variables

o Nominal variables

o Ordinal variables

• Nominal variables

o Nominal variables allow for only qualitative classification.

o That is, they can be measured only in terms of whether the

individual items belong to certain distinct categories, but we cannot

quantify or even rank order the categories

o Nominal data has no order, and the assignment of numbers to

categories is purely arbitrary.

o Because of lack of order or equal intervals, one cannot perform

arithmetic (+, -, / or *) or logical operation (<, >, =) on the nominal

data.

o E.g. male and female, unmarried, married, divorce or widower.

• Ordinal variables

o A discrete ordinal variable is a nominal variable, but its different

states are ordered in a meaningful sequence

o Ordinal data has order, but the intervals between scale points may

be uneven.

o Because of lack of equal distances, arithmetic operations are

impossible, but logical operations can be performed on the ordinal

data.

o A typical example of an ordinal variable is socio-economic status of

families.

o We know upper middle is higher than middle but we cannot say how

much higher.

o Ordinal variables are quite useful for subjective assessment of

quality; importance or relevance.

o Ordinal scale data are very frequently used in social and behavioral

research.

Page 105: Dr+Saiful's+Notes+on+Medical+%26+Allied+Health+Education+-+Statistics+%26+Research+Methodology

104

o Almost al opinion surveys today request answers on three-, five- or

seven-point scale.

o Such data are not appropriate for analysis by classical techniques,

because the numbers are comparable only in terms of relative

magnitude, not actual magnitude.

o Consider for example a questionnaire item on the time involvement

by selecting one of the following codes:

1 = very low or nil

2 = low

3 = medium

4 = great

5 = very great

4. Response variables/target variables

• Often called a dependent variable or predicted variable.

• This is the variable that is being watched and/or measured

5. Explanatory variables/predictor variables

• Any variable that explains the response variable or predictor variable.

• Its values will be used to predict the value of the target variable.

• This is the variable manipulated by the experimenter.

6. Confounding variable

• A confounding variable (also confounding factor, lurking variable, a

confound, or confounder) is an extraneous variable in a statistical

Ratio absolute zero

Interval distance is meaningful

Ordinal attributes can be ordered

Nominal attributes are only name; weakest

Page 106: Dr+Saiful's+Notes+on+Medical+%26+Allied+Health+Education+-+Statistics+%26+Research+Methodology

105

model that correlates (positively or negatively) with both the

dependent variable and the independent variable.

• Extraneous variables are undesirable variables that influence the

relationship between the variables that an experimenter is examining.

• In other words, confounding is a variable that is associated with the

predictor variable and is a cause of the outcome variable.

Page 107: Dr+Saiful's+Notes+on+Medical+%26+Allied+Health+Education+-+Statistics+%26+Research+Methodology

106

DATA PRESENTATION

1. Two ways of presenting data

• Tables

• Charts

2. Tables

• One-way table (Univariate)

o Table 1: Number of respondents by gender

Gender No. of respondents

Male 51

Female 49

Total 100

• Two-way table (Bivariate)

o Table 2: Number of respondents by gender and their educational

qualification

Gender Primary Secondary Higher Total

Male 15 20 16 51

Female 14 20 12 49

Total 29 40 38 100

Gender Primary (%) Secondary (%) Higher (%) Total (%)

Male 15 ( ) 20 ( ) 16 ( ) 51 ( )

Female 14 ( ) 20 ( ) 12 ( ) 49 ( )

Total 29 ( ) 40 ( ) 38 ( ) 100 ( )

Page 108: Dr+Saiful's+Notes+on+Medical+%26+Allied+Health+Education+-+Statistics+%26+Research+Methodology

107

Gender Primary

(%)

Secondary

(%)

Higher

(%)

Total

(%)

Male 15

( )

20

( )

16

( )

51

( )

Female 14

( )

20

( )

12

( )

49

( )

Total 29

( )

40

( )

38

( )

100

( )

3. Charts

• Charts is a graphically way to organize data

• Types

o Pie chart

A pie chart is a graphical way to organize data

All pie charts compare parts of a whole

A lie chart uses percentages of fraction to compare data

A type of graph in which percentages values are represented as

proportionally-sized slices of a pie

Pie charts are especially useful in representing proportions,

percents and fractions.

o Bar chart and Histogram

A histogram is a bar graph that shows that frequency data

The first step… collect data and sort it into categories

Label the data as the independent set or the dependent set

Data group would be the independent variable and the frequency

of that set would be the dependent variable

The horizontal axis should be label with independent variable

The vertical axis should be labeled with the dependent variable

Each mark on either axis should be equal increments, such as 2,

4, 6, 8, etc

I think histogram as “sorting bin”

Page 109: Dr+Saiful's+Notes+on+Medical+%26+Allied+Health+Education+-+Statistics+%26+Research+Methodology

108

You have one variable, and you sort data by this variable by

placing them into “bins”

Then you count how many pieces of data are in each bin

The height of the rectangle you draw on top each bin is

proportional to the number of pieces in that bin

On the other hand, in bar graph you have several measurement

of different items, and compare them

The main question a histogram is “how many measurements are

there in each of the classes of measurement?”

The main question a bar graph answer “what is the measurement

for each item?”

Situation Bar graph or Histogram?

We want to compare total revenues of

five different companies

Bar graph.

Key question: what is the revenue for

each company?

We have measured revenues of several

companies. We want to compare

numbers of companies that make from

0 to 10,000; from 10,000 to 20,000;

from 20,000 to 30,000 and so on

Histogram.

Key question: how many companies

are there in each class of revenue?

We want to compare height of ten oak

tree in a city park

Bar graph

Key question: what is the height of

each tree?

We have measured several trees in a

city park. We want to compare

numbers of trees that are from 0 to 5

meters high; from 5 to 10; from 10 to

15 and so on

Histogram

Key question: how many trees are

there in each class of height?

o Line graph

Are more popular than all other graphs combined because their

visual characteristics reveal data trends clearly and these graphs

are easy to create

Page 110: Dr+Saiful's+Notes+on+Medical+%26+Allied+Health+Education+-+Statistics+%26+Research+Methodology

109

A line graph is a visual comparison of how two variables – shown

on the x- and y-axis – are related or vary with each other.

It shows related information by drawing a continuous line

between all the points on a grid.

Line graphs compare two variables: one is plotted along the x-axis

(horizontal) and the other along the y-axis (vertical)

The y-axis is a line graph usually indicates quantity (e.g. dollars,

liters) or percentage, while the horizontal x-axis often measures

units of time.

o Scattered plot

The pattern of the data points on the scatter plot reveals the

relationship between the variables.

Scatter plots can illustrate various patterns and relationship,

such as:

• Data correlation

• Positive or direct relationships between variables

• Negative or inverse relationship between variables

• Scattered data points

• Non-linear patterns

• Spread of data

• outliers

o Pictograph

Page 111: Dr+Saiful's+Notes+on+Medical+%26+Allied+Health+Education+-+Statistics+%26+Research+Methodology

110

Z-Score & IT’S USES

1. Z-score serves 2 purposes

• Each z-score will tell the exact location of the original x value within the

distribution

• The z-score will form a standardized distribution that can be directly

compared to other distributions that also have been transformed into z-

scores.

2. Value of z-score

• The sign tells whether the score is located above (+) or below (-) the

mean

• The number tells the distance between the score and the mean in terms

of the number of standard deviation.

0.3413

In every normal distribution 0.3413 of its total area lies between the mean and z = 1.2

This is the formula for converting a given value of x into its corresponding z score for raw data:

x = the value that is being standardized μ = the mean of the distribution σ = standard deviation of the distribution

Standard Error formula:

σ = standard deviation n = sample size

Z-score for Means

= sample mean = standard error

= population mean

Page 112: Dr+Saiful's+Notes+on+Medical+%26+Allied+Health+Education+-+Statistics+%26+Research+Methodology

111

• The z-score for an item, indicated how far and in what direction, that

item deviates from its distribution’s mean, expressed in units of its

distribution’s standard deviation.

• The mathematics of the z score transformation are such that if every

item in a distribution is converted to its z score, the transformed scores

will necessarily have a mean of zero and a standard deviation and a

standard deviation of one.

• Z scores are sometimes called “standard scores”.

• The z score transformation is especially useful when seeking to

compare the relative standings of items from distributions with

different standard deviations.

• Z scores are especially informative when the distribution to which they

refer, is normal.

• In every normal distribution, the distance between the mean and a

given z score cuts off a fixed proportion of the total area under the

curve.

3. Z-score for making comparison

• For example: bob receive a score of x = 60 on math exam and a score x

= 56 on a biology test. For which course he did well?

o Suppose the biology score had µ = 48 and = 4 and the math score

had µ = 50 and = 10.

o Suppose you use a test for your students and the µ = 65 and = 10

and your friend use a test for your students which have µ = 100 and

= 15

o Three of your students got 75, 45 and 67 respectively in your test

what should be the score of your students in your friend test if you

want to say the students’ performance in both the tests are same

o Formula for standardized score is x = µ + z

• Second example: Ho = there is no effect of PBL on average score

obtained by the students

o Average score (µ) of USM 3rd year students is 60 with standard

deviation () is 5

Page 113: Dr+Saiful's+Notes+on+Medical+%26+Allied+Health+Education+-+Statistics+%26+Research+Methodology

112

o A sample of 20 students attended PBL and average score of this

group of students is 65

o Is this increase of 5 marks in average due to chance or the effect of

PBL?

o Answer can be obtained by z test

Ho of the students attended PBL = 60

H1 of students attended PBL > 60 or ≠ 60

Level of significance or level = 0.05 (usually used)

Z = 1.96

Z = [sample mean – hypothetical mean]/[standard error between and µ]

Z = [obtained difference]/[difference due to chance]

Consult normal distribution table to see if calculated value is in the critical

region or not to reject or accept null hypothesis

= sample mean = standard error

= population mean

Standard Error formula:

σ = standard deviation n = sample size

Page 114: Dr+Saiful's+Notes+on+Medical+%26+Allied+Health+Education+-+Statistics+%26+Research+Methodology

113

t-test

1. In calculating z-score we need

• µ = population mean

• = population standard deviation

• When the standard deviation () is not known, t-test is the alternative.

2. In simple t-test instead of , sample variance is used.

• Sample variance (S2) = [SS/n-1] = [SS/df]

o SS = Σ x2 – ( [Σx]2/n )

SS = Sum of squared deviation

3. Instead of standard error x, estimated standard error Sx is used.

• Estimated standard error Sx = S/(√n) = √(S2/n)

• t = [X - µ]/Sx

o X = sample mean

o µ = population mean (hypothesis mean)

o Sx = estimated standard error from sample

• The higher the the degree of freedom (df) (sample size) the closure the

S2 (sample variance) to the 2 (population variance)

• Example:

o I prefer PBL than Lecture

Response 1 = SA, 2 = A, 3 = UD, 4 = DA, 5 = SDA

From this example hypothesis mean (µ) = 3

But µ can be getting from study that has been done by someone

previously.

4. Independent measures t-test

• t = [ (X1 – X2) – (µ1 - µ2) ]/ (standard error)

• Pooled variance, S2p = [ SS1 + SS2 ]/[ df1 + df2 ]

• Two samples standard error, Sx1 – Sx2 = √[ (S2p/n1) + (S2p/n1) ]

• Ho = there is no difference in the m=clinical performance of students

attended traditional curriculum and PBL curriculum.

Page 115: Dr+Saiful's+Notes+on+Medical+%26+Allied+Health+Education+-+Statistics+%26+Research+Methodology

114

SENSITIVITY & SPECIFICITY

1. Definition

• Sensitivity

o Proportion of subject with a target condition who are identified by a

positive test finding.

o Test’s ability to correctly identify individuals with the condition

o Test’s capacity to detect the condition when it is truly present

o Probability of a test being positive given that the condition is present

o Also called true positive rate or hit rate

o The test will actually classify a person (with the condition) as likely

to have the condition

• Specificity

o Proportion of subjects free of the condition who are correctly

identified by a negative test result

o Test’s ability to correctly identify individuals without the condition

o Test’s capacity to exclude condition when it is truly absent

o Also called true negative rate or correct rejection rate

o The test will actually classify a person (without the condition) as

unlikely to have the condition

Respondents With the

condition

Without the

condition Total

With the condition a 36 b 96 132

Without the condition c 4 d 864 868

Total 40 960 1000

Sensitivity = a/(a+c) = .90 true positive

Specificity = d/(b+d) = .90 true negative

Positive Predictive Power (PPP) = a/(a+b) = .27 false positive

Negative Predictive Power (NPP) = d/(c+d) = .99 false negative

Page 116: Dr+Saiful's+Notes+on+Medical+%26+Allied+Health+Education+-+Statistics+%26+Research+Methodology

115

2. Validity of the Test

True status (population)

positive negative

positive a b Result

of test negative c d

Sensitivity: the probability of testing positive if the condition is truly

present = a/(a+c)

Specificity: the probability of screening negative if the condition is truly

absent = d/(b+d)

Example: Screening breast cancer by Physical Exam & Mammography

Respondents With the

condition

Without the

condition Total

With the condition a 36 b 96 132

Without the condition c 4 d 864 868

Total 40 960 1000

Sensitivity: a/(a+c)

= 36/(36+4)

= 0.90 = 90%

Interpretation screening by physical exam and mammography will

identify 90% of all true breast cancer cases

Specificity: d/(b+d)

= 864/(96+864)

= 0.90 = 90%

Interpretation screening by physical exam and mammography will

correctly classify 90% of all non-breast cancer patient as being free disease.

PPP = a/(a+b)

= 36/(36/96)

Page 117: Dr+Saiful's+Notes+on+Medical+%26+Allied+Health+Education+-+Statistics+%26+Research+Methodology

116

= 0.27 = 27%

NPP = d/(c+d)

= 864/(864 + 4)

= 0.99 = 99%

Validity – the extend to which the test distinguishes between persons with

and without the condition

High validity require

- High sensitivity

- High specificity